WaBis

walter.bislins.ch

Kann etwas schneller sein als Lichtgeschwindigkeit?

Dienstag, 21. Februar 2012 - 17:14 | Autor: wabis | Themen: Wissen, Physik, Kosmologie | Kommentare(53)

Kannst du mir sagen, ob sich etwas schneller fortbewegen kann als Lichtgeschwindigkeit? Nach Einstein geht das ja nicht, aber der ist - glaub ich - nun überholt, oder? Es würde mich interessieren, ob man sich im Raum auch schneller fortbewegen kann. Sonst hätte man ja Jahre bis zu einem fernen Planeten.

Überlichtgeschwindigkeit ist durch die Gleichungen der Relativitätstheorie nicht kategorisch ausgeschlossen. Lediglich das Überschreiten der Lichtgeschwindigkeit ist nicht möglich. Theoretisch könnten sich Teilchen (sog. Tachyonen) mit Überlichtgeschwindigkeit bewegen. Diese hätten aber Eigenschaften, die komplett verrückt anmuten. So könnten sie z.B. nicht unter Lichtgeschwindigkeit fallen, hätten eine imaginäre Ruhemasse und würden bei Energieabgabe, zum Beispiel durch Stahlung, sogar noch schneller werden! Betrachtet man Tachyonen jedoch quantenmechanisch, stellt man fest, dass sich selbst diese als lokale Störung nicht überlichtschnell ausbreiten können. Dazu mehr später.

Das OPERA-Experiment

Ich nehme an, du hast vom sog. OPERA-Experiment gehört, wo Neutrinos schneller als das Licht eine mehr als 700km lange Strecke durch die Erde zurückgelegt haben sollen. Die Neutrinos sind dabei mit 1,0000248-facher Lichtgeschwindigkeit durch den Berg geflogen und damit ca. 60 ns (60 milliardstel Sekunden) eher eingetroffen als das Licht. Der gemessene Effekt ist also extrem klein. Trotzdem ist das Resultat nach einer genauen Fehleranalyse statistisch relevant! [1]

Lange wurde angenommen, dass Neutrinos keine Masse haben und sich daher mit Lichtgeschwindigkeit bewegen. Alles, was keine Masse besitzt (z.B. Licht und Gravitation) bewegt sich im Vakuum immer mit Lichtgeschwindigkeit. Dies war auch in Übereinstimmung mit dem sehr erfolgreichen Standardmodell der Elementarteilchenphysik. Neuere Experimente haben aber gezeigt, dass Neutrinos eine kleine Masse haben [1]. Daher dürften sie nie ganz Lichtgeschwindigkeit erreichen.

Die Tatsache, dass Neutrinos eine kleine Masse haben, bedeutet, dass das Standardmodell erweitert werden muss. Es ist eine Eigenschaft, die nicht ins bisherige Bild der Quantenmechanik passt. Dass jetzt aber Neutrinos sogar schneller als das Licht sein sollen, passt überhaupt nicht ins Bild. Es gibt mehrere Theorien, die das versuchen zu erklären. Jede dieser Theorien macht bestimmte Vorhersagen über die Eigenschaften von Neutrinos, welche noch experimentell bestätigt werden müssen. Die meisten Physiker sind der Meinung, dass irgendwo ein Fehler oder unerwarteter Effekt im sehr komplizierten Experiment versteckt ist. Bisherige Messungungen der Neutrinogeschwindigkeit von Supernovas zeigten im Rahmen der Messgenauigkeit keine Abweichung von der Lichtgeschwindigkeit. [1]

Aber mal von Neutrinos abgesehen: Alles, was eine Masse hat, kann die Lichtgeschwindigkeit nicht überschreiten. Was keine Masse hat (Photonen und ein paar andere masselose Bosonen) bewegen sich immer mit Lichtgeschwindigkeit im Vakuum. In Materie ist die Lichtgeschwindigkeit übrigens kleiner als im Vakuum. Tatsächlich gibt es sogar die Möglichkeit, dass Materieteilchen in bestimmten Materialien schneller sind als das Licht in diesem Material!

Wieso kann Materie nicht schneller als Licht werden?

Wenn man einen Körper, der in Ruhe ist, in Bewegung versetzen will, braucht man eine Kraft , die ihn beschleunigt, z.B. einen Raketenmotor. Je grösser die Masse des Körpers ist, umso grösser muss auch diese Kraft sein. Je länger die Kraft wirkt, umso schneller wird der Körper. Sobald die Kraft weggenommen wird, behält der Körper seine Geschwindigkeit bei, wenn er nicht durch Reibung abgebremst wird. Die Geschwindigkeit , die ein Körper nach einer gewissen Zeit hat, kann mit folgender Formel nach Newton berechnet werden:

(1)

Auf beiden Seiten integrieren ergibt:

(2)

Wenn wir die Kraft als konstant annehmen, vereinfacht sich die Formel (2) zu:

(3)
wobei'
' =' 'Geschwindigkeit nach der Zeit
' =' 'Konstante Kraft mit der das Objekt beschleunigt wird
' =' 'Masse des Objektes
' =' 'Zeitdauer der Beschleunigung

Nach dieser Formel kann jede beliebige Geschwindigkeit erreicht werden, auch Lichtgeschwindigkeit und mehr. muss nur genügend gross sein und genügend lange wirken. Würde man z.B. eine Masse von 1 kg mit einer Kraft von 10 Newton beschleunigen, würde man nach obiger Formel in weniger als einem Jahr (347 Tage) die Lichtgeschwindigkeit von 300'000 km/s überschreiten.

Aber da macht uns die Spezielle Relativitätstheorie von Einstein einen Strich durch die Rechnung:

Relativistische Masse verhindert Lichtgeschwindigkeit

Die Masse eines Körpers ist nicht konstant, sondern hängt von seiner Geschwindigkeit ab! Das was wir geläufig als Masse bezeichnen ist immer die sog. Ruhemasse . Solange wir uns wesentlich langsamer als das Licht bewegen, ist die relativistische Masse nur unwesentlich grösser als die Ruhemasse und somit können wir in einem solchen Fall die Newton-Formeln verwenden. Wenn die Geschwindigkeit aber in Bereiche der Lichtgeschwindigkeit kommt, wird die relativistische Masse immer grösser und grösser nach folgender Formel aus der Speziellen Relativitätstheorie:

(4)
wobei'
' =' 'Relativistische Masse des Objektes
' =' 'Ruhemasse des Objektes
' =' 'Geschwindigkeit des Objektes
' =' 'Lichtgeschwindigkeit ca. 300'000 km/s

Wenn sehr viel kleiner als ist, dann ist noch viel kleiner als und somit viel viel kleiner als 1, fast Null. Die obige Formel wird dann zu:

(5)

Wir sehen damit, dass für alle uns gewohnten Geschwindigkeiten die obige Formel (4) zu degradiert. Das heisst, wir spüren immer nur die Ruhemasse eines Körpers.

Wenn nun aber die Geschwindigkeit z.B. 1/2 Lichtgeschwindigkeit ist, so ergibt sich:

(6)

Mit 1/2 Lichtgeschwindigkeit ist z.B. eine Ruhemasse von 1 kg schon 155 Gramm schwerer. Das scheint nicht viel. Setzen wir für aber z.B. 90% der Lichtgeschwindigkeit ein:

(7)

Bei dieser Geschwindigkeit ist die Ruhemasse von 1 kg schon 2.29 kg schwer! Nehmen wir mal 99% Lichtgeschwindigkeit:

(8)

Bei dieser Geschwindigkeit ist die Ruhemasse von 1 kg schon 7.07 kg schwer! Nehmen wir mal 99.99 % Lichtgeschwindigkeit:

(9)

Und so weiter. Im Extremfall, wenn ist, erhalten wir für die Masse :

(10)

Je grösser die Masse ist, umso mehr Kraft muss man aufwenden, um sie weiter zu beschleunigen. Dabei wird mit zunehmender Geschwindigkeit immer grösser und grösser. Bei Lichtgeschwindigkeit würde unendlich gross und man müsste eine unendlich grosse Kraft aufwenden, um die Geschwindigkeit weiter zu erhöhen.

Dies ist also der Grund, weshalb nichts, was eine Masse hat, jemals die Lichtgeschwindigkeit erreichen oder gar überschreiten kann.

In Teilchenbeschleunigern wird dies täglich bewiesen. Dort werden Elementarteilchen wie Protonen oder Elektronen bis auf annähernd Lichtgeschwindigkeit beschleunigt. Ihre relativistische Masse nimmt dabei stark zu und es muss immer mehr Energie aufgewendet werden, die Teilchen auf einer Kreisbahn zu halten und weiter zu beschleunigen.

Betrachtungen bei Überlichtgeschwindigkeit

Nehmen wir mal an, es gäbe Teilchen, die mit Überlichtgeschwindigkeit fliegen. Wie gross wäre dann ihre relativistische Masse nach obiger Formel? Nehmen wir mal , also doppelte Lichtgeschwindigkeit:

(11)

Ups! Was ist die Wurzel von -3? Das ergibt keine reelle Zahl. Es gibt nur eine imaginäre Lösung (wenn man mit komplexen Zahlen rechnet). Daher müssten also Tachyonen eine imaginäre Masse haben, was immer das sein soll. Da Energie ist, müssten solche Teilchen auch eine imaginäre Energie haben. Daher würden solche Teilchen sogar noch schneller, wenn sie Energie abstrahlen könnten. Eine komplett verkehrte Welt.

Nicht lokale Geschwindigkeiten können grösser als Lichtgeschwindigkeit werden

Es gibt aber dennoch Geschwindigkeiten, die grösser sind als die Lichtgeschwindigkeit! Wie du sicher weisst, dehnt sich das Universum gleichmässig, d.h. in alle Richtungen gleich aus. Das bedeutet, dass sich Galaxien umso schneller von uns weg bewegen, je weiter entfernt sie von uns sind. Tatsächlich gibt es eine Entfernung, ab der sich Galaxien mit Überlichtgeschwindigkeit von uns entfernen! Dies ist aber kein Widerspruch zu Einsteins Theorie. Denn tatsächlich bewegen sich die Galaxien überhaupt nicht oder im Vergleich zur Lichtgeschwindigkeit unbedeutend langsam. Aber der Raum selbst, also das Vakuum in dem die Galaxien schweben, bewegt sich dort mit Überlichtgeschwindigkeit von uns weg! Es entsteht so immer mehr Vakuum und die Vakuum-Energie (auch als Dunkle Energie bezeichnet) bewirkt die Expansion des Universums.

Diese Tatsache hat eine unheimliche Konsequenz: Jede Galaxie, welche diese Grenze überschreitet, an der sie sich mit Lichtgeschwindigkeit von uns entfernt, ist für uns für alle Zeiten unerreichbar! Es kann kein Signal oder Licht von dort jemals wieder zu uns gelangen! Man spricht bei dieser Grenze von einem Ereignishorizont, weil er dieselben Eigenschaften hat wie der Ereignishorizont von Schwarzen Löchern. Man schätzt, dass das ganze Universum viele Grössenordnungen mal grösser als das für uns sichtbare Universum ist. Das sichtbare Universum hat einen Radius von ca. 13.7 milliarden Lichtjahre und das ist nur ein winzig kleiner Teil des ganzen Universums!

Exponentielle Expansion des Universums

Aber es kommt noch verrückter: Das Universum dehnt sich nicht konstant mit derselben Rate aus, sondern die Ausdehnungsrate nimmt exponentiell zu! In einigen Milliarden Jahren haben alle Galaxien, die nicht zur lokalen Gruppen (unsere Milschstrasse, Andromeda und ein paar duzend weitere nahe Galaxien) gehören den Ereignishorizont überschritten. Dann können wir statt der geschätzten 100 milliarden Galaxien, die man jetzt sehen kann, nur noch ein paar lokalen Galaxien sehen, wobei sich unsere Milchstrasse dann mit Andomeda vereinigt haben wird.

Wir befinden uns in einer Zeit, in der die Expansion von einer Verlangsamung gerade in eine Beschleunigung geändert hat. Es ist die astronomisch wohl vielfältigste und interessanteste Zeit, die sich die Menschheit ausgesucht hat.

Sind Sternenreisen möglich?

Um zu anderen Sternen reisen zu können hält die Relativitätstheorie eine Hintertür offen: Je schneller man fliegt, umso langsamer vergeht die Zeit und umso kürzer werden Strecken in Flugrichtung. Für ein Lichtteilchen, das sich mit Lichtgeschwindigkeit bewegt, bleibt die Zeit stehen und das Universum ist für dieses Teilchen zu einer Scheibe mit Null Dicke geschrumpft. Es ist auf seiner Flugbahn quasi überall gleichzeitig!

Wenn wir nur schnell genug fliegen, können wir auch tausende von Lichtjahren entfernte Sterne zu Lebzeiten erreichen. Das Problem ist nur, dass für die Daheimgebliebenen unterdessen viele tausend Jahre vergehen und wir mit ihnen nicht mehr Kommunizieren könnten.

Im folgenden Blog-Beitrag kannst du konkrete Berechnungen anstellen lassen:

 Raketenflug Einstein gegen Newton

Weitere Informationen

OPERA-Neutrino-Anomalie; Wikipedia
https://de.wikipedia.org/wiki/OPERA%2DNeutrino%2DAnomalie

Kommentare

1Heiri 05.04.2013 | 21:44

Hoi Walti,

ich habe deinen Blog mit grossem Interesse gelesen. Vorerst sind mir 2 Dinge nicht ganz verständlich:

F: Neutrinogeschwindigkeit Opera

Nach der Korrektur wurden offenbar keine Abweichungen von Neutrinogeschwindigkeit und Lichtgeschwindigkeit festgestellt. Auch von Supernova-Ereignissen – die Neutrinos sind ja wirklich sehr lange unterwegs gewesen - seien keine Unterschiede gemessen worden.
Heisst das nicht, dass Neutrinos keine Masse haben?

F: Lichtgeschwindigkeit, Längenkontraktion

Ich will vorausschicken, dass ich in keiner Weise an der Relativitätstheorie zweifle. Ich möchte lediglich erfahren, ob ich mir das richtig vorstelle.

Gedankenexperiment:

Ich habe ein Raumschiff, das unbegrenzt Energie hat und in der Lage ist, konstant mit 1 g zu beschleunigen. Ich gedenke, zum Sirius zu reisen (8,5 LJ), bis in die Hälfte beschleunigen und dann bremsen, immer mit 1 g.

Die Reise verläuft planmässig, nach 5,7 Jahren (wenn ich richtig gerechnet habe) komme ich am Ziel an. Jetzt kommt da einer und sagt, dass die LG die höchste Geschwindigkeit sei. Ich entgegne, dass ich ja 8,5 LJ in 5,7 Jahren zurückgelegt habe, also müsse ich erheblich schneller als die LG unterwegs gewesen sein.

Natürlich weiss ich, dass für den stationären Beobachter erheblich mehr Zeit vergangen ist.
Müsste man die Aussage: "Nichts kann schneller als LG sein" nicht relativieren, etwa "Im System des Beobachters ist die höchste Geschweindigkeit die LG".
Oder anders gesagt, wie kann eine Geschwindigkeit absolut sein, wenn die Zeit relativ ist?

Beste Grüsse
Heiri

2wabiswalter@bislins.ch (Walter Bislin, Autor dieser Seite) 08.04.2013 | 02:58

A: Neutrinogeschwindigkeit Opera

Hallo Heiri

Zur ersten Frage habe ich einen Blog-Beitrag geschrieben:

 Masse und Geschwindigkeit von Neutrinos

3wabiswalter@bislins.ch (Walter Bislin, Autor dieser Seite) 10.04.2013 | 01:11

A: Lichtgeschwindigkeit, Längenkontraktion

Hallo Heiri, hier noch die Antwort auf Deine zweite Frage:

Mein Programm kommt auf folgende Werte:

  • Strecke von der Erde aus gesehen: d = 8,5 Lj
  • Flugzeit von der Erde aus gesehen: T = 10,26 Jahre
  • Flugzeit aus Sicht der Rakete: τ = 4,59 Jahre
  • Strecke aus Sicht der Rakete: S = 3,262 Lj!

Du hast übersehen, dass für die Rakete nicht nur die Zeit langsamer läuft, sondern auch die Distanz entsprechend schrumpft (siehe Längenkontraktion).

Die Rakete muss also nicht 8,5 Lj sondern nur 3,262 Lj zurücklegen, wofür die Raketenzeit von 4,59 Jahre locker reicht, ohne die Lichtgeschwindigkeit überschreiten zu müssen!

4Heiri 12.04.2013 | 21:44

Vielen Dank Walti, auch für Deine Antwort bezüglich der Neutrino-Masse.

F: Problem mit der Distanz und der Lichtgeschwindigkeit

Längenkontraktion. Ich bin verwirrt. Meine Vorstellung: Wenn ich Protonen im CERN auf fast c beschleunige, sehe ich von aussen das Proton immer flacher werden. Der Beobachter im Proton würde davon aber nichts bemerken. Wenn ich dieses nun auf 0 abbremsen würde, hätte es doch trotzdem x Runden à y km zurückgelegt.

Die Distanz zum Sirius bleibt doch erhalten, oder nicht? Und da für den Raumfahrer nur 4,6 Jahre vergangen sind, müsste er schliessen, dass er, mindestens aus seiner Sicht, c überschritten hat. Wo ist mein Denkfehler?
Mit freundlichen Grüssen
Heiri

5Georg 12.04.2013 | 22:40

Hallo Heiri, das habe ich auch immer gedacht.

Das Raumschiff startet und beschleunigt mit 1 g, bis zur Hälfte der Strecke, dann bremst es wieder mit 1 g ab, bis es angekommen ist. Das Licht hat es während des Fluges immer überholt. Das Raumschiff kommt nach 10,26 Jahren an, das Licht braucht nur 8,5 Jahre. Die Differenz beträgt 1,76 Jahre. Wenn es Überlichtgeschwindigkeit fliegen würde, müsste die Differenz negativ werden, also Rückwärts in der Zeit und das geht nicht.

Nehmen wir an, dass alle 0,01 Jahre, von der Erde aus ein Lichtimpuls gesendet wird. Im Raumschiff hätte man 1026 Impulse gezählt. Nur die Zeitdifferenzen zwischen den Impulsen hätten die Raumfahrer im Schiff verkürzt wahrgenommen. Auch hier, keine Überlichtgeschwindigkeit.

Georg

6Heiri 13.04.2013 | 11:35

Hallo Georg,

Da hast Du natürlich Recht! Aber: Die Uhr im Raumschiff zeigt 4,59 Jahre an. Wenn der Astronaut die Geschwindigkeit ausrechnet, erhält er 1,85 LJ pro Jahr, denn die Distanz zur Erde ist ja immernoch 8,5 LJ (was ihm der Wissenschaftsoffizier Spok jun auch bestätigte).
Ich möchte damit eigentlich nur sagen, dass die LG nicht soooo absolut ist, sondern auch vom Beobachter abhängt.

Beste Grüsse
Heiri

7wabiswalter@bislins.ch (Walter Bislin, Autor dieser Seite) 14.04.2013 | 01:14

A: Problem mit der Distanz und der Lichtgeschwindigkeit

Heiri schrieb: Die Distanz zum Sirius bleibt doch erhalten, oder nicht? Und da für den Raumfahrer nur 4,6 Jahre vergangen sind, müsste er schliessen, dass er, mindestens aus seiner Sicht, c überschritten hat. Wo ist mein Denkfehler?

Hallo Heiri

Du darfst nicht Werte aus einem System mit Werten des anderen Systems mischen!

System Erde:

  • Reisestrecke: d = 8,5 Lj
  • Flugzeit: T = 10,26 J
  • ergibt eine Durchschnittsgeschwindigkeit von v = d / T = 0,828 c

System Rakete:

  • Reisestrecke wegen Längenkontraktion: s = 3,262 Lj
  • Flugzeit wegen Zeitdilation: τ = 4,59 j
  • ergibt eine Durchschnittsgeschwindigkeit von v' = s / τ = 0,711 c

In beiden Systemen wird die Lichtgeschwindigkeit nie überschritten!

Achtung: Würde die Rakete nicht beschleunigen sondern den ganzen Weg mit konstanter Geschwindigkeit fliegen, wären die beiden errechneten Geschwindigkeiten in beiden Systemen gleich gross!

Ich verstehe, dass du Probleme damit hast, dass im Erde-System die Distanz 8,5 Lj ist, im Raketen-System dieselbe Distanz aber nur 3,262 Lj sein soll. Wie ist das möglich, dass ein und dieselbe Distanz von der Rakete aus gesehen kürzer sein soll?

Dies hat mit der Zeitdehnung zu tun! Für die Astronauten vergeht ja die Zeit langsamer als auf der Erde. Nehmen wir als Beispiel mal an, die Zeit vergehe gerade halb so schnell wie auf der Erde. Und nehmen wir eine Messstrecke von der Länge s (von der Erde aus gemessen). Nehmen wir an, von der Erde aus gesehen brauche die Rakete 2 Sekunden um die Strecke s zu passieren.

Wie stellt sich diese Situation in der Rakete dar?

Da die Zeit in der Rakete nur halb so schnell vergeht wie auf der Erde, vergeht in der Rakete während dem Passieren der Strecke s nur 1 Sekunde. Die relative Geschwindigkeit zwischen Erde und Rakete ist aber von beiden Systemen aus gemessen gleich gross (mit umgekehrter Richtung). Das kann aber nur bedeuten, dass die Strecke s von der Rakete aus gemessen nur halb so lange scheint, wie sie wirklich ist! Dieser Effekt wird als Längenkontraktion bezeichnet. Für die Astronauten scheint die Strecke nicht nur zu schrumpfen, sondern sie schrumpft tatsächlich, denn sie brauchen nur noch die halbe Zeit um die Strecke in ihrer Zeit zu passieren. Für sie ist die Längenkontraktion nicht einfach nur ein Rechenergebnis, sondern realität!

Fliegt eine Rakete so schnell, dass die Zeit in der Rakete gegenüber der Erde nur halb so schnell vergeht, so messen die Astronauten für eine Reise von 2 Lichtjahren eine Reisezeit von nur 1 Jahr. Für sie hat sich die Reisestrecke halbiert, denn sie fliegen ja mit einer Geschwindigkeit, in welcher sie 2 Jahre für diese Strecke brauchen müssten (von der Erde aus gesehen). Sie erreichen ihr Ziel mit dieser Geschwindigkeit aber bereits in einem Jahr ihrer Lebenszeit.

Die Effekte der speziellen Relativitätstheorie sind symmetrisch. Von der Erde aus gesehen schrumpf die Länge der Rakete mit zunehmender Geschwindigkeit. Die Astronauten ruhen in ihrem System. Das bedeutet, dass der Weltraum mit der Reisegeschwindikeit an ihnen vorbeizieht. Damit schrumpf für die Astronauten die Erde, die Reisestrecke, ja der ganze Weltraum in Flugrichtung!

Ich hoffe, ich habe mich verständlich machen können.

8wabiswalter@bislins.ch (Walter Bislin, Autor dieser Seite) 14.04.2013 | 03:09

A: Rechenbeispiel mit unbeschleunigter Rakete

Ich möchte mal ein einfaches Beispiel durchrechnen, um ein Gefühl für die spezielle Relativitätstheorie zu vermitteln.

Gegeben:

  • Distanz zu einem anderen Planeten: d = 12 Lj
  • Reisegeschwindigkeit: v = 12/13 c

Daraus ergibt sich eine Reisezeit von der Erde aus gesehen von:

(12)

Die Reisezeit für die Astronauten berechnet sich wiefolgt:

(13)

Die Astronauten legen also in 5 Jahren ihrer Zeitrechnung eine Strecke von 12 Lichtjahren zurück. Aber halt! 12 Lichtjahre in 5 Jahren? Fliegen die Astronauten aus ihrer Sicht mit Überlichtgeschwindigkeit?

Nein! Denn die Astronauten dürfen nicht die Distanz von der Erde aus gemessen durch ihre Eigenzeit teilen um die Geschwindigkeit in ihrem System zu messen. In ihrem System schrumpft die Reiselänge wegen der Längenkontraktion auf die Länge :

(14)

Damit kann die Geschwindigkeit im Raketen-System berechnet werden:

(15)

Damit erhalten wir für denselben Wert wie für . In beiden Systemen wird dieselbe Geschwindigkeit gemessen, jedoch mit unterschiedlich langen Strecken und Zeiten!

Man sieht ausserdem, dass weder im Erde-System noch im Raketen-System die Lichtgeschwindigkeit überschritten wird.

Frage: Die beiden Systeme können sich offenbar nur über ihre Geschwindigkeiten einigen. Weder Zeit noch Distanzen scheinen in beiden Systemen gleich zu sein. Wie hängen denn die beiden Systeme zusammen? Was haben sie ausser der Geschwindigkeit noch gemeinsam?

Antwort: Das Raumzeit-Intervall!

Das Raumzeit-Intervall ist, wenn wir nur eine Raumrichtung betrachten, wiefolgt definiert:

(16)
wobei'
' =' 'Raumzeit-Intervall
' =' 'Zeitunterschied zwischen zwei Ereignissen im Ruhesystem
' =' 'Distanz zwischen zwei Ereignissen im Ruhesystem
' =' 'Zeitunterschied zwischen denselben zwei Ereignissen vom bewegten System aus gesehen
' =' 'Distanz zwischen denselben zwei Ereignissen vom bewegten System aus gesehen
' =' 'Lichtgeschwindigkeit in Lj/j

oder einfacher ausgedückt:

(17)

Ein Raumzeit-Intervall setzt sich zusammen aus zwei Ereignissen. Ein Ereignis hat immer eine Position und eine Zeit. Position und Zeit beziehen sich immer auf ein bestimmtes Inertialsystem. In jedem Inertialsystem kann der zeitliche und räumliche Abstand zweier Ereignisse unterschiedlich sein, das daraus gebildete Intervall ist aber in allen Inertialsystemen gleich!

Beispiel

Das erste Ereignis in unserem Beispiel ist der Start der Rakete von der Erde an der Position zur Zeit . Das zweite Ereignis ist die Ankunft der Rakete am Ziel mit und der Zeit . Der räumliche Abstand ist somit und der zeitliche Abstand .

Das entsprechende Intervall ist:

(18)

Jetzt begeben wir uns ins Intertialsystem der Rakete. Bezüglich des mit der Rakete mitfliegenden Inertialsystems bleibt die Rakete während der ganzen Reise in Ruhe. Damit ist der räumliche Abstand . Die Rakete braucht für die Reise von den Astronauten aus gesehen 5 Jahre, also ist der zeitliche Abstand .

Damit ist das Raumzeit-Intervall der Rakete :

(19)

Wir sehen, dass die beiden Raumzeit-Intervalle und gleich sind!

Über das Raumzeit-Intervall kann man einfach die Reisezeit aus Sicht der Astronauten berechnen:

(20)

Bemerkung: in obigen Formeln muss für die Lichtgeschwindigkeit folgendes eingesetzt werden:

(21)

Denn das Licht legt eine Distanz von einem Lichtjahr pro Jahr zurück.

9bernhard 10.10.2013 | 16:00

F: Überlichtgeschwindigkeit nachgewiesen?

Bei 3 Dingen wurde bereits Überlichtgeschwindigkeit im Vakuum also 299 792 458 m/s nachgewiesen:

1. Beim "Superluminaren Tunneln" mit Photonen wurde bereits 4,7 fache Lichtgeschwindigkeit nachgewiesen.Allerdings war das Signal am Tunnelende nur noch sehr schwach.

2. Bei der "Quantenverschränkug" besteht eine Art kommunikation zwischen 2 Elektronen egal wie weit sie vonneinander entfernt sind.Sehr warscheinlich verläuft es ohne Zeitdifferenz das heisst dass die Geschwindigkeit unendlich ist.

3. Die Ausdehnung des Universums. Fremde Galaxien bewegens sich von uns weg und zwar immer schneller um so weiter sie weg sind. Die Geschwindigkeit ist 74 km/(s·Mpc) (Mpc = Megaparsec), dabei wird aber nur der Raum bewegt

10wabiswalter@bislins.ch (Walter Bislin, Autor dieser Seite) 10.10.2013 | 18:13

A: Überlichtgeschwindigkeit nachgewiesen?

@bernhard

Dazu kann ich nur auf einen Artikel der Wikipedia verweisen, der zu all diesen Phänomenen neutral Stellung nimmt:

 Artikel Überlichtgeschwindigkeit in der Wikipedia

11Heiri 24.11.2013 | 00:22

F: Längenkontraktion bei Lichtgeschwindigkeit

Hallo Walter,

Ich habe immer noch Schwierigkeiten mit der Längenkontraktion.

Wie ich es sehe:

Die Längen in Fahrtrichtung der Gegenstände im Raumschiff und das Raumschiff selbst erscheinen einem ausserhalb des Raumschiff „stillstehenden“ Beobachters verkürzt.

Ist diese Längenkontrakton auch gültig ausserhalb des Raumschiffs? Das System Sonne – Sirius ist ja in Ruhe.
Wäre das eine Distanzkontraktion? Google sagt zu diesem Stichwort wenig bis nichts.

Nun zum Ausdruck „Distanz aus Sicht des Raumschiffes“:

Erster Versuch:

Nehmen wir einmal an, von der Sonne zum Sirius wäre ein Schnur gespannt, an der all 1 Mio km eine Lampe wäre. Der Kapitän würde während der Reise die Lampen zählen und die Zeiten (mit Borduhr) zwischen den Lampen messen.

Anhand dieser Messungen könnte er ein Distanz-Geschwindigkeitsdiagramm erstellen. Würde dieses von der Planung abweichen, die ohne die Berücksichtigung der RT erstellt wurde?

Zweiter Versuch:

Er montiert eine laaaange Stange senkrecht zur Fahrtrichtung derart, dass das Raumschiff in der Mitte ist. Ein optisches Distanzmessgerät. Mittels Triangulation misst er laufend die Distanzen zur Sonne und zum Sirius. Die Addition beider Distanzen müsste entweder stets 8,5 LJ ergeben oder aber bis zur Mitte immer kleiner und nachher wieder grösser werden.

Meine Idee: Der Raumfahrer hat keine Möglichkeit, die „Distanzkontraktion“ festzustellen.

Vielen Dank!

12Mischa 08.01.2015 | 12:51

F: Längenkontraktion für Licht

Wie sieht den die Längenkontraktion bei Lichtgeschwindigkeit aus? Da würde die Strecke nach obiger Formel auf 0 verkürzt werden, und das Licht wäre überall gleichzeitig?

13wabiswalter@bislins.ch (Walter Bislin, Autor dieser Seite) 08.01.2015 | 13:42

@Mischa

A: Längenkontraktion für Licht

Genau! Du hast es erfasst! Das ist die spezielle Rolle die die Lichtgeschwindigkeit spielt in der SR. Dadurch dass das Licht überall gleichzeitig ist, von sich aus gesehen, vereint es Raum und Zeit: Bei Lichtgeschwindigkeit schrumpfen Raum und Zeit zu Null. Das heisst das Raumzeit-Intervall von Licht ist immer Null.

Weil Raum und Zeit nach der SR die Raumzeit bilden, also nicht mehr unabhängige Grössen sind, erscheint die Lichtgeschwindigkeit für alle Beobachter die gleiche, egal wie schnell sie sich bewegen. Das gilt auch umgekehrt: Weil die Lichtgeschwindigkeit für alle Beobachter dieselbe ist, muss gelten, dass Raum und Zeit keine voneinander unabhängigen Grössen sind, was mit dem Begriff Raumzeit ausgedrückt wird.

14Mischa 14.01.2015 | 15:30

F: Licht und Raumkrümmung

Ich hätte da noch eine frage betreffend der Raumkrümmung....

Man kann z.B. Bei einer Sonnenfinsterniss einen Stern sehen, welcher sich hinter der Sonne befindet, da sich das Licht entlang der Raumkrümmung bewegt.

Wie kann das Licht auf einen Körper treffen, wenn es sich ja entlang der Raumkrümmung um den Körper bewegt? Teilt sich das Licht?

Und was ist an der Stelle wo der Raum gekrümmt wird? Nichts?

Diese Frage lässt mich nicht in Ruhe.

15wabiswalter@bislins.ch (Walter Bislin, Autor dieser Seite) 10.04.2017 | 23:03

A: Licht und Raumkrümmung

Hi Mischa. Die folgende Grafik zeigt Dir wie das Licht eines Sternes, der hinter der Sonne steht, sich ausbreitet und dabei teilweise um die Sonne gebogen wird. Jener gebogene Lichtstrahl (dicke orange Linie), welcher trotz Krümmung genau die Erde trifft, gibt uns die scheinbare Position des Sterne an. Im rechten Teil des Bildes siehst Du, wie das von der Erde aus aussieht.

Der 5-zackige Stern stellt die wirkliche Position des Sternes dar. Der 4-zackige Stern stellt die scheinbare Position des Sternes dar.

Lichtkrümmung um eine Sonne

16St.Manfred 17.04.2015 | 18:58

F: Warpfelder

Hier noch ein interessanter Artikel zum Thema Reisen mit "Überlichtgeschwindigkeit" (gerade weil hier mittels Warp Feld nicht die Gesetze der Physik ausgehebelt werden):

http://en.wikipedia.org/wiki/White%E2%80%93Juday_warp-field_interferometer

Leider gab es von Dr. Harold White in letzter Zeit keine bahnbrechenden Neuigkeiten in Bezug auf seine Forschung ...

17wabiswalter@bislins.ch (Walter Bislin, Autor dieser Seite) 17.04.2015 | 19:26

A: Warpfelder

@St.Manfred

Raumzeitkrümmung (also eine Art Warpfeld?) kann im Prinzip mit jeder Art von Masse oder Energie erzeugt werden. Aber wenn es schon die Masse einer Sonne braucht, um Licht nur einen Bruchteil einer Bogensekunde zu beugen, dann kann man verstehen, dass in einem Labor entsprechende Messungen unterhalb des Messbaren liegen.

Ich sehe noch weitere theoretische Probleme, die mit den heute bekannten Gesetzen der Physik nicht gelöst werden können:

  • Wie kann man auf der einen Seite des Raumschiffs eine positive Krümmung und auf der anderen eine negative Krümmung der Raumzeit erzielen? Für negative Krümmungen braucht es negative Energie!
  • Die benötige Energiemenge, um auch nur ein wenig nutzbare Raumzeitkrümmung zu erreichen, ist derart enorm, dass dazu nicht mal Materie/Antimaterie hilfreich ist. Die komprimierteste Form von Energie ist ja Materie-Masse. Es braucht ja schliesslich auch schon einen Planeten der Grösse der Erde um nur schon 1 g Beschleunigung zu erreichen. Da hilft auch Anitmaterie nicht, denn es spielt für die Raumzeitkrümmung keine Rolle, ob die Energie in Form von Masse oder Photonen oder sonst was vorliegt.

Siehe auch die Diskussionsseite zu obigem Artikel in der Wikipedia:

http://en.wikipedia.org/wiki/Talk:White%E2%80%93Juday_warp-field_interferometer

18Max Jugl 11.11.2015 | 14:55

Sehr geehrter Verfasser, oder eben guten Tag Heiri,
Zunächst einmal hat mir Ihr Artikeln außerordentlich gut gefallen!! Genau das, was ich für meine Vorwissenschaftliche Arbeit brauche (bin 16 Jahre alt)!!
Nun stellte sich mir doch noch eine Frage:
mr = m0 / Wurzel -3 kann ich ja nicht mit den herkömmlichen rationalen Zahlen berechnen!! Deswegen brauche ich die komplexen Zahlen!! Jetzt wollte ich Sie fragen wie das dann aussieht?? Wie setze ich hier i ein??
Heißt das: mr = m0 / (i * Wurzel 3)!!?? Und was bedeutet das dann?? Kann ich umformen und sagen die Ruhemasse der tachyonen ist deshalb imaginäre wegen:
m0 = mr * i * Wurzel 3??
Keine Ahnung!! Hoffe sehr, Sie können es mir erklären!! Wäre total wichtig!! Ich Danke Ihnen für Ihre Zeit und wünschen Ihnrn noch einen schönen Tag!!

Mit freundlichen Grüßen,

Max Jugl

19Tom 27.09.2016 | 21:32

F: Relativistische Masse

Hat die relativistische Masse Einfluss auf die Gravitation?

20wabiswalter@bislins.ch (Walter Bislin, Autor dieser Seite) 30.09.2016 | 23:44

A: Relativistische Masse

@Tom: Soviel habe ich herausgefunden:

Hier meine Übersetzung der Antwort zur Frage does a moving object curve space time as its velocity increases?, beantwortet von John Rennie am 24.1.2014 auf physics.stackexchange.com:

Nein, relativistische Masse verändert die Raumzeitkrümmung nicht, gravitiert also nicht.

Betrachten wir das Gravitationsfeld der Sonne. Wir sind effektiv stationär im Vergleich zur Sonne, weil unsere relative Bewegung viel kleiner als Lichtgeschwindigkeit ist. Die Sonne selbst rotiert weit unter relativistischen Geschwindigkeiten. Also können wir erwarten, dass ihr Gravitationsfeld durch die Schwarzschild-Metrik beschrieben werden kann.

Der Metrik-Tensor ist in Bezug auf Koordinatentransformationen invariant. Also würde ein Beobachter, der mit fast Lichtgeschwindigkeit an der Sonne vorbei fliegt, auch feststellen, dass die Gravitation um die Sonne mit der Schwarzschild-Metrik beschrieben werden kann. Die Metrik schaut nicht gleich aus im Koordinatensystem des Beobachters, d.h. die gij Koordinaten wären verschieden, aber es ist derselbe Tensor. Weil der Beobachter in seinem Koordinatensystem stationär ist und die Sonne an ihm vorbei fliegt, ist die Schlussfolgerung, dass die Geschwindigkeit die Raumzeit-Krümmung nicht ändert.

Nebenbei erwähnt, das ist auch der Grund, warum ein schnelles Objekt sich nicht in ein Schwarzes Loch verwandelt.

21Meel 27.10.2016 | 17:56

Q: Kann ein Neutron Licht überholen?

Eine Frage:
Wenn ein sehr hochgeladenes Neutron in einen massereichen Gegenstandt (Glas) fliegt und bei 99.9999 Prozent der lichtgeschwindigkeit ist und ein Photon neben ihm einen sehr geringen Abstandt herausgeflogen ist. Wenn diese beiden dann auf den Gegenstandt treffen heisst das nicht dass das Neutron das Photon "überholt".

22wabiswalter@bislins.ch (Walter Bislin, Autor dieser Seite) 30.10.2016 | 16:07

A: Kann ein Neutron Licht überholen?

Wie schnell sich Licht, also Photonen, in einem Material bewegt wird durch den Brechungsindex des Materials angegeben: Geschwindigkeit im Material = Lichtgeschwindigkeit / Brechungsindex.

Für verschiedene Wellenlängen des Lichtes ist der Brechungsindex in der Regel unterschiedlich. Daher wird weisses Licht, das eine Überlagerung von Photonen der verschiedensten Wellenlängen ist, in einem Prisma aufgeteilt in seine Grundfarben.

Ist nun der Brechungsindex von Licht hoch genug, kann sich ein Neutron durchaus schneller in einem Material bewegen als Licht und somit Licht in diesem Material überholen. Heute gibt es Materialien, welche die Geschwindigkeit von Licht bis auf wenige Meter pro Sekunde "bremsen" können! Da sind dann sogar relativ langsame Neutronen schneller als das Licht in diesem Material.

Die Antwort lautet also: Ja, schnelle Neutronen können Licht in bestimmten Materialien überholen.

Beachte, dass das Licht innerhalb des Materials trotzdem die Lichtgeschwindigkeit c hat. Es interagiert jedoch mit den Atomen, indem es bildlich gesprochen absorbiert und verzögert wieder emitiert wird. Das Licht macht also auf seinem Weg immer wieder Pausen, sodass die Durchnittsgeschwindigkeit kleiner als c wird. Das ist ein vereinfachtes Bild. Physikalisch und mathematisch ist das viel komplizierter, kommt aber auf dasselbe heraus.

23René Hübscher 07.12.2016 | 12:03

Q: Überlichtgeschwindigkeit mit unelastischen Körpern?

Folgendes Gedankenexperiment:

Ein Planet ist eine Lichtminute von der Erde entfernt. Zwischen der Erde und dem Planeten ist ein dünnes Rohr verlegt, welches komplett mit Kugeln gefüllt ist. An beiden Enden des Rohres steht eine Person. Die Person auf der Erde schickt gleichzeitig ein Lichtsignal zum anderen Planeten und drückt eine weitere Kugel in das Rohr. Das Lichtsignal ist erst nach einer Minute auf dem anderen Planeten sichtbar (Entfernung 1 Lichtminute), die Kugel am Ende des Rohres fällt aber praktisch sofort aus dem Rohr. Eine einzelne Kugel hat sich natürlich sehr langsam, und nur um den Kugelumfang im Rohr bewegt. Aber die Information, dass eine Kugel in das Rohr geschoben wurde, ist praktisch sofort am anderen Ende erkennbar, da dort sogleich eine Kugel aus dem Rohr fällt.

Was ist falsch an diesem Gedankenexperiment?
Danke und Gruss
René

24SevenOfNine 07.12.2016 | 15:01

A: Information mit Überlichtgeschwindigkeit?
@René: Du hast sogar 2 Stolpersteine im Experiment:
1.) Der kleine Stolperstein - eine Kugel bewegt sich im Rohr jeweils nur um ihren Durchmesser, wenn hinten eine neue Kugel nachgeschoben wird. Teile Deinen Kugelumfang durch 3.1415927 um zum Durchmesser zu gelangen....:-)
2.) Der grosse Stolperstein - Der Vergleich zw. Lichtstrahl und Kugelrohr hat im wahrsten Sinn ungleich lange Spiesse, denn die herausfallende Kugel ist ja schon (fast) auf dem anderen Planeten; Ein Lichtstrahl hingegen muss die gesamte Distanz zurücklegen. Angebrachter wäre eine "Taschenlampe", welche ebenfalls fast auf dem anderen Planeten liegt und von der Erde aus mit einer bereits verlegten Schnur durch Ziehen eingeschaltet würde.
Konklusion: die Information trifft bei gleich langen Spiessen auch gleichzeitig ein.
Gruss von den Borg

25Florian Sorea 08.12.2016 | 00:47

A/F: Information mit Überlichtgeschwindigkeit?

Ich finde das Gedankenexperiment von Rene Hübscher sehr interessant und ich zerbreche mir grad den Kopf darüber :)

Wenn man genügend Kraft aufwenden könnte, um alle Kugeln entlang der Lichtminute zu bewegen, würde dann nicht die hinterste Kugel gleichzeitig um einen Kugel-Durchmesser bewegt wie die am Anfang der Kette? Natürlich wären die Kugeln nicht schnell unterwegs, aber man könnte dann ja mit Überlichtgeschwindigkeit kommunizieren zwischen den beiden Planeten? (zB wenn man die Kugeln in Morse-Abständen einschieben würde?)
ODER: Wenn man zwischen den beiden Planeten einen nicht elastischen Draht spannen würde`? Auf Planet A wird daran gezogen(mit genügend Kraft). Bewegt sich die Schnur sofort auf Planet B?

Oder braucht die Kraftübertragung durch die Materie der Kugeln/ des Drahtes Zeit?
Wie kann man sich das vorstellen? Falls sich diese Information auch mit LG bewegt, müsste ja, sobald man die Kugel einschiebt, 1min lang eine Kugel zuviel im Rohr sein? bzw beim Bsp 2, 1min lang der Draht um die Gezogene Länge länger sein? :D

Man könnte das Gedanken-Experiment auch mit einem Kugelstosspendel machen, wo ich mir das Ganze noch eher vorstellen kann.

Wenn man das Pendel zwischen diesen beiden Planeten aufstellen würde (in entsprechender Länge), und auf Planet A eine Kugel anheben und auf die Kugelreihe prallen lassen würde, würde die letzte Kugel in der Kette auf Planet B sofort weggestossen? Oder ist diese Übertragung ebenfalls mit Lichtgeschwindigkeit unterwegs? Das könnte ich mir noch eher vorstellen :) Nur bewegen sich ja die einzelnen Kugeln entlang der Kette in diesem Beispiel nicht.

26wabiswalter@bislins.ch (Walter Bislin, Autor dieser Seite) 10.12.2016 | 13:39

A: Überlichtgeschwindigkeit mit unelastischen Körpern?

Gäbe es perfekte unelastische, steife Körper wäre Kommunikation dieser Art mit Überlichtgeschwindigkeit möglich. In der Praxis pflanzt sich der Stoss oder Zug mit der Schallgeschwindigkeit vS in diesem Material fort. Diese Geschwindigkeit ist abhängig von der Steifigkeit E und der Dichte ρ (Gewicht pro Volumen) des Materials:

(22)

Je steifer das Material umso höher die Geschwindigkeit, je dichter umso niedriger. Im Prinzip könnte man sich ein Material vorstellen, dessen Eigenschaften so sind, dass vS grösser als die Lichtgeschwindigeit c wird. Die Relativitätstheorie verbietet aber solche Stoffe. Sie gibt eine absolute obere Grenze für die Steifigkeit eines Materials vor, sodass die Lichtgeschwindigkei c nicht überschritten wird:

(23)

Die transversale Schallgeschwindigkeit in Eisen beträgt 5170 m/s. Lichtgeschwindigkeit ist grob 300 000 000 m/s. Der Stoss pflanzt sich also ca. 58 000 mal langsamer fort als Licht.

Man muss sich einen echten Stoff wiefolgt vorstellen: Er besteht aus lauter Atomen, die durch elektronagnetische Kräfte zusammengehalten werden. Ein benachbartes Atorm wird von einer Bewegung eines Atoms über Änderungen im elektromagnetischen Feld informiert. Diese Information kann sich maximal mit Lichtgeschwindigkeit fortpflanzen. Bei realen Stoffen kann man sich die Atome als Massekügelchen und die Verbindungen als Federn vorstellen. Je leichter die Kügelchen und je steifer die Federn, umso schneller pflanzt sich eine Welle im Material fort. Um das zu berechnen, muss die Wellengleichung des Modells aufgestellt und gelöst werden. Das Resultat ist dann die obige Formel (22).

Einen Stoss kann man sich so vorstellen:

Impuls in Solid Material

Quellen

27René Hübscher 21.12.2016 | 17:24

Vielen Dank für die Erklärungen. Natürlich meinte ich den Durchmesser und nicht den Umfang. War so fasziniert vom Gedanken, dass ich mit diesem Detail sehr grosszügig umging :-)

Wenn ich die Ausführungen zu unelastischen Körpern wieder zurück in unsere erfahrbare Umwelt transformiere, müsste dies ja bedeuten:
Vor mir liegt ein 1 Meter langer Ast. Ich trete gegen diesen Ast, so dass diese sich in Längsrichtung um einen Meter von mir weg verschiebt. Das würde dann ja bedeuten dass der Ast während dem Verschieben kürzer ist als ein Meter. Wenn das Ende, an welches ich getreten habe, zum Stillstand kommt, bewegt sich das gegenüberliegende (vordere) Ende noch weiter, bis der Stock seine ursprüngliche Länge wieder erlangt hat. Die beiden Enden kommen folglich nicht gleichzeitig zum Stillstand. Ist diese Schlussfolgerung richtig? .

28wabiswalter@bislins.ch (Walter Bislin, Autor dieser Seite) 23.12.2016 | 20:49

Das hängt von der Steifigkeit des Objektes ab. Ein Ast ist jedoch so steif, dass folgendes passiert:

  • Du trittst gegen das eine Ende des Astes -> der Ast wird an diesem Ende ein wenig komprimiert
  • Die Komprimierung pflanzt sich schnell dem Ast entlang zum anderen Ende -> Der Ast ist solange leicht verkürzt
  • Die Stauchung kommt am anderen Ende -> der Ast hat wieder seine ursprüngliche Länge
  • Der Ast bewegt sich nun als Ganzes eine bestimmte Strecke fort, während seine Länge der Anfangslänge entspricht.
  • Der Ast wird als Ganzes wegen der Reibung am Boden langsamer, bis er zum Stillstand kommt.

Das heisst also, dass der Ast während des Verschiebens NICHT kürzer ist bis er zum Stillstand kommt, sondern nur einen ganz kurzen Augenblick lang, bis sich der Stoss durch den Ast ans andere Ende verschoben hat. Solange sich der Ast danach bewegt wird er in seiner ursprünglichen Länge als Ganzes verschoben.

Nur eine Beschleunigung an einem Ende erzeugt eine Kompressionswelle, die dem Ast entlang zum anderen Ende wandert. Hat der Ast mal seine Geschwindigkeit erreicht, ist die Beschleunigung 0 und der Ast behält die Ursprüngliche Länge bei. Da er wegen der Reibung als ganzes Abgebremst wird, entsteht dabei keine Kompressionswelle mehr. Der Ast ist also nur einen kurzen Moment während dem Kick etwas kürzer.

29ramue 17.03.2017 | 20:21

Sie erklären komplizierte physikalische Vorgänge sehr gut mit Formeln und auch mit Worten und Vergleichen.
Vielen Dank für diese Seite.
Ich habe zwei Fragen auf die Tante Google zwar Ergebnisse liefert, ich die Antworten aber nicht verstehe.
(Außerdem landet man dann schnell bei "Einstein-Leugnern" und das bin ich bestimmt nicht :-)

Hintergrund (soweit von mir verstanden)
Masse behaftete Teilchen bewegen sich langsamer als Lichtgeschwindigkeit im Vakuum (c)
Photonen sind masselos und bewegen sich im Vakuum mit (c)
Photonen sind das "Austauschteilchen" (="Kraftübertrager") des elektromagnetischen Feldes
Die Energie des Photon "steckt" in der Frequenz des Photons (E=h*f)
Photonen mit der höchsten Energie sind Gamma-Photonen und "Höhenstrahlung" ( f > 10^21 Hz)

Frage 1: Was hindert die Photonen sich im Vakuum schneller als c zu bewegen? Was "bremst" denn da?

Frage 2: Gibt es eine Begrenzung der Energie von Photonen, sprich eine höchste Frequenz, die nicht überschritten werden kann? Wenn ja, wie wird diese Grenze definiert?

Vielen Dank für ihre Mühe!

30wabiswalter@bislins.ch (Walter Bislin, Autor dieser Seite) 10.04.2017 | 16:44

Hallo Ramue

Hier meine Antworten auf Deine Fragen:

Lichtgeschwindigkeit als Höchstgeschwindigkeit

Nach dem Verständnis, das uns die Spezielle Relativitätstheorie gibt, ist die Frage "was bremst denn da?" etwa so sinnvoll wie die Frage "warum kann man nicht nördlicher als bis zum Nordpool gehen?". So wie letzteres eine Eigenschaft einer Kugel ist, ist die Tatsache, dass nichts schneller als die Lichtgeschwindigkeit sein kann, eine Eigenschaft der Raumzeit.

Die Lichtgeschwindigkeit hat eigentlich nichts mit Licht zu tun. Es ist die Höchstgeschwindigkeit, mit der sich Kausalität (Abfolge aufeinander bezogener Ereignisse und Zustände) ausbreitet. Und diese Geschwindigkeit ist für alle Beobachter dieselbe, egal wie schnell sie sich bewegen. Man sagt, diese Geschwindigkeit ist Invariant.

Die Relativitätstheorie gibt uns die Lorentz-Transformation, mit der man physikalische Grössen, wie Zeiten, Längen, Geschwindigkeiten und andere davon abgeleitete Grössen, zwischen verschiedenen unbeschleunigten Koordinatensystemen umrechnen kann. Eine dieser Grössen ist die relativistische Energie eines Teilchens:

(24)
mit
wobei'
' =' 'relativistische Energie
' =' 'Masse des Teilchens
' =' ' = relativistischer Impuls des Teilchens mit Masse ungleich Null
' =' ' = nicht-relativistischer (klassischer) Impuls des Teilchens mit Masse ungleich Null
' =' 'Geschwindigkeit des Teilchens bezüglich dem Beobachter
' =' 'Lichtgeschwindigkeit

Der blaue Term entspricht der Ruhe-Energie . Der rote Term enstpricht dem Impuls oder der Bewegungsenergie. Der grüne Term ist die Zusammenfassung der Formel für Teilchen mit Masse. Bei masselosen Teilchen bleibt nur der rote Term, also der Impuls mal c übrig. Die Formel (E_relativistisch) folgt aus der Lorentz-Transformation, also dass Raum und Zeit keine unabhängigen Dinge sind, sondern verschiedene Projektionen der Raumzeit, und daher miteinander verknüpft sind.

Wenn die Geschwindigkeit v gegen Lichtgeschwindigkeit c geht, geht gegen Unendlich. Das heisst, dass bei konstanter Masse m die Energie wegen gegen Unendlich gehen muss. Mit anderen Worten, man muss unendlich viel Energie aufbringen, um ein Teilchen, das eine Masse m0 hat, auf Lichtgeschwindigkeit zu beschleunigen.

Wenn nun ein Teilchen eine bestimmte Energie hat, seine Masse aber gegen Null geht, muss gegen Unendlich gehen, damit seine Energie nicht Null wird. Das heisst also, dass die Geschwindigkeit eines Teilchens mit verschwindend kleiner Masse, zum Beispiel eines Neutrinos, gegen Lichtgeschwindigkeit gehen muss. Es leuchtet ein, dass im Grenzfall m = 0 die Geschwindigkeit v gleich Lichtgeschwindigkeit c sein muss.

Lichtgeschwindigkeit als Konsequenz der Lorentz-Transformation

Wenn wir die Flugbahn eines Teilchens in einem Bezugssystem aufzeichnen, können wir auch ausrechnen, wie diese Flugbahn aus der Sicht eines anderen dazu gleichförmig bewegten Bezugssystems aussieht. Solche Umrechnungen nennt man Transformationen. Diese Transformationen kann man auch auf alle physikalischen Gesetze anwenden, nicht nur auf Flugbahnen. Die in der speziellen Relativitätstheorie verwendete Transformation nennt man die Lorentz-Transformation.

Wir können beobachten, dass in jedem unbeschleunigten Bezugssystem, ein solches nennt man Inertialsystem (IS), dieselben physikalischen Gesetze gelten. Es gibt kein spezielles IS, das still steht. Es gibt nur relative Bewegungen. Diese Eigenschaften bezeichnet man als das Relativitätsprinzip. Weiterhin stellen wir fest, dass die physikalischen Gesetze nicht von der Bewegungsrichtung abhängig sind (Isotropie) und in jedem Teil des Weltraumes gleich sind (Homogenität).

Eine Koordinaten-Transformation zwischen Inertialsystemen kann alleine aus grundsätzlichen Annahmen wie Homogenität, Isotropie und dem Relativitätsprinzip hergeleitet werden [1]. In dieser Herleitung der Transformation erscheint ein konstanter invarianter Parameter k, der unabhängig von der Geschwindigkeit v ist:

(25)

Allgemeine Transformtion zwischen IS

wobei'
' =' 'Zeit und Position gemessen in einem IS
' =' 'Zeit und Position desselben Ereignisses gemessen in einem anderen IS
' =' 'relative Geschwindigkeit der beiden IS zueinander
' =' 'invarianter Parameter mit der Einheit 1 / (Geschwindigkeit im Quadrat)

Untersuchen wir mal, was dieser Parameter k für eine Rolle spielt:

Negative Werte für k führen zu Quadratwurzeln aus negativen Zahlen, was in Realität keinen Sinn macht. Also kann k nicht negativ sein.

Wir können k auch durch eine andere Konstante ausdrücken, die für die folgenden Überlegungen praktischer ist: k = 1 / c2, wobei c die Einheit einer Geschwindigkeit hat. Beachte, dass c wie k nicht von der Relativgeschwindigkeit v der beiden IS abhängig ist. Es handelt sich also um einen sog. invarianten Wert, der in allen IS exakt denselben Wert hat.

Wenn wir für c = einsetzen, was einem Wert k = 0 entspricht, reduziert sich die Transformation auf die Galilei-Transformation, mit Galilei-Newtonscher Kinematik, unendlicher Lichtgeschwindigkeit, absolutem Raum und absoluter Zeit.

Wenn wir für c irgendeinen positiven Wert ungleich Null und ungleich unendlich wählen, was einem Wert k > 0 enstpricht, erhalten wir die Lorentz-Transformation:

(26)

Lorentz-Transformation

Aus dieser Transformtion folgt, dass c die grösst mögliche Geschwindigkeit ist, die in irgendeinem IS auftreten kann, denn für v > c wird der Ausdruck unter der Wurzel negativ und die Wurzel hat keine reelle Lösung. Zudem ist c für alle IS gleich, also nicht von v abhängig (invariant). Erwähnenswert ist zudem, dass für Geschwindigkeiten v << c (v ist viel kleiner als c) die Lorentz-Transformation in die Galilei-Transformation übergeht.

(27)

Galilei-Transformation

Für Geschwindigkeiten v weit unter der Lichtgeschwindigkeit sind daher die klassischen Formeln von Newton für die Beschreibung von Bewegungen genau genug. Dies gilt jedoch nicht für den Elektromagnetismus. Dort muss immer mit relativistischen Formeln gerechnet werden.

Ole Rømer war der erste, der feststellte, dass die Lichtgeschwindigkeit nicht unendlich ist. James Clerk Maxwell fand durch Experimente zwei Konstanten, eine für die Magnetische Permeabilität μo und eine für die Permittivität (dielektrische Leitfähigkeit) εo, die zusammen die Ausbreitungsgeschwindigkeit c = 1 / √μo·εo für elektromagnetische Wellen ergaben. Diese Konstanten sind unabhängig von irgend einer Geschwindigkeit, haben also in jedem IS denselben Wert - sie sind invariant. Maxwell erkannte, dass c die von Ole Rømer gemessene Geschwindigkeit für Licht ist. Das bedeutete also, dass Licht eine elektromagnetische Welle sein musste.

Die Tatsache, dass es in unserem Universum invariante Grössen gibt, bedeutet, dass die Lorentz-Transformation zwischen zwei IS die korrekte Transformation sein muss. Die gefundene invariante Geschwindigkeit c für Licht bedeutet, dass dieses c das invariante c der Lorentz-Transformation sein muss. Dieses c muss also die grösst mögliche Geschwindigkeit für physikalische Effekte sein. Aus der Lorentz-Transformation kann abgeleitet werden (wie oben gezeigt), dass alle masselosen Teilchen mit dieser Geschwindigkeit unterwegs sein müssen. Es ist zudem auch die Geschwindigkeit, mit der sich kausale Zusammenhänge fortpflanzen, da sich alle physikalischen Effekte mit maximal c im Vakuum fortpflanzen können.

Aus der Lorentz-Transformation folgen dann alle weiteren Erscheinungen wie Zeitverlangsamung (Time Dilation), Längenkontraktion, relativität der Gleichzeitigkeit, E = m·c2 usw. Alle diese Vorhersagen haben sich in unzähligen Experimenten und Anwendungen (siehe z.B. GPS, Teilchenbeschleuniger, PET-Scanner) als real erwiesen.

Warum es in unserem Universum eine invariante Höchstgeschwindigkeit gibt, und keine unendlich hohe, bleibt eine offene Frage. Gao's "Vermutung" ist, dass die invariante Höchstgeschwindigkeit mit der Quantisierung von Raum-Zeit bei der sog. Plank-Scale zusammenhängen könnte [2].

Sicher ist nur, dass es bei einer unendlich hohen Lichtgeschwindigkeit keinen Elektromagnetismus in der uns bekannten Form gäbe, folglich auch keine Atome. Und damit wären auch wir nicht existent.

Rapidität des Lichtes

Bei relativistischen Geschwindigkeiten dürfen verschiedene Geschwindigkeiten nicht einfach addiert werden. Sonst würde man Summen erhalten können, die höher als Lichtgeschwindigkeit sind. Wenn die Lichtgeschwindigkeit unendlich wäre, könnte man Geschwindigkeiten einfach addieren.

Vielleicht messen wir nicht die richtige Art Geschwindigkeit?

Die Lorentz-Transformation kann auch symmetrisch durch hyperbolische Rotation geschrieben werden:

(28)

Der Parameter der hyperbolischen Funktionen und stellt die Geschwindigkeit zwischen den Inertialsystemen dar, jedoch in anderen Einheiten. Man nennt diese Geschwindigkeit Rapidität. Sie ist definiert als:

(29)

Die Rapidität hat die Eigenschaft, dass sie Werte zwischen und + annehmen kann und dass solche Geschwindigkeiten einfach addiert werden können. Wenn wir v = c setzen wird die Rapidität ω = . Die Lichtgeschwindigkeit in Einheiten der Rapidität ist also Unendlich.

Quellen und weiterer Lesestoff:

Derivation of the Lorentz Transformations; Wikipedia(en)
https://en.wikipedia.org/wiki/Derivations%5Fof%5Fthe%5FLorentz%5Ftransformations%23From%5Fgroup%5Fpostulates
Relativity without light: A further suggestion; paper by Shan Gao
http://www.academia.edu/2293371/Relativity_without_light_A_further_suggestion
[3]
Why does the speed of light c have the value it does?
http://physics.stackexchange.com/questions/56973/why-does-the-speed-of-light-c-have-the-value-it-does
[4]
Why is the speed of light the upper limit rather than the speed of particle t?
http://physics.stackexchange.com/questions/15135/why-is-the-speed-of-light-the-upper-limit-rather-than-the-speed-of-particle-t
[5]
Why is there a universal speed limit?
http://physics.stackexchange.com/questions/205222/why-is-there-a-universal-speed-limit

Maximale Energie von Photonen

Frage 2: Gibt es eine Begrenzung der Energie von Photonen, sprich eine höchste Frequenz, die nicht überschritten werden kann? Wenn ja, wie wird diese Grenze definiert?

Für Photonen gibt es im Prinzip keine minimale und keine maximale Energie. Man kann immer Bezugssysteme (IS) finden, von denen aus gesehen ein Photon eine beliebige Energie haben kann. Angenommen in unserem Bezugssystem hat ein Photon die Energie Eγ. In einem Bezugssystem, das sich auf diese Photon zu bewegt, ist die Wellenlänge gestaucht, also blau-verschoben, und damit die Energie des Photons grösser nach der Formel:

(30)
wobei'
' =' 'Energie des Photons von einem IS aus gesehen
' =' '6,626 ·10−34 J·s = Plancksches Wirkungsquantum
' =' '299 792 458 m/s = Lichtgeschwindigkeit im Vakuum
' =' 'Wellenlänge des Photons

Umgekehrt erscheint die Wellenlänge aus einem Bezugssystem, das sich vom Photon weg bewegt, gedehnt, also rot-verschoben, und damit ist seine Energie in diesem Bezugssystem kleiner. Weil die Geschwindigkeit zwischen zwei Bezugssystemen innerhalb der Grenze Lichtgeschwindigkeit beliebig sein kann, gibt es auch keine prinzipielle Grenze für die Energie eines Photons.

Die niedrigste Energie eines beobachtbaren Photons hat eine Wellenlänge vom Durchmesser des beobachtbaren Universums. Dies ergibt eine minimale Energie von ca. 2,3 ·10−52 J. Als Wellenlänge habe ich den Durchmesser des heute des beobachtbaren Universums von 93,2 Milliarden Lichtjahre angenommen.

Man weiss jedoch heut noch nicht, ob es eine obere Grenze der Energie eines Photons gibt. Wenn nämlich die Energie eines Photons hoch genug wird, müssen Effekte der allgemeinen Relativitätstheorie berücksichtigt werden. Uns fehlt jedoch noch eine Theorie, welche Quantenmechanik und allgemeine Relativitätstheorie vereinigt. Da Masse und Energie äquivalent sind, könnte man annehmen, dass bei genügend hoher Energie ein schwarzes Loch (in der Grösse einer Planck-Einheit?) entstehen könnte.

Die Energie eines solchen Photons wäre weit höher als alles, was bisher beobachtet worden ist. Ein solches Photon hätte eine Wellenlänge im Bereich der Planck-Länge 1,616 ·10−35 m und damit eine Energie von 12,3 ·109 J. Zum Vergleich: Als Gammastrahlung werden Photonen mit Wellenlängen kürzer als 5 ·10−12 m bezeichnet. Das entspricht Energien pro Photon grösser als 4 ·10−14 J. Das sind bis zu 23 Grössenordnungen Unterschied zu hochenergetischer Gamma-Strahlung.

Quellen:

[1]
Are there any theoretical limits on the energy of a photon?
http://physics.stackexchange.com/questions/159922/are-there-any-theoretical-limits-on-the-energy-of-a-photon
31Ralf Müller 15.04.2017 | 16:42

Vielen Dank für die Erklärungen!

32Werner Schneider 27.10.2017 | 09:12

Die Randbedigungen der Lichtgeschwindigkeit, die gut erklärt sind, verführen leider zu der störrischen Auffassung man müsse die Lichtgeswindigkeit als Herausforderung eines Grenzübertritts verstehen und eine Geschwindigkeit sukzesive von Nul bis Unendlich steigern.

Das ist eindeutig verfehlt, denn die Lichtgeschwindigkeit wird von keiner Masse mit m(0)>0 erreicht.
Nur die Quanten des Elektromagnetsimus nehmen diese Geschwindigkeit auf - auch noch mit der Besonderheit
dass nur eine Freisetzungs-Energie am Ort der Quelle genügt, diese Lichtschnelligkeit starr zu übernehmen, wie unter einem vorgegebenen Zwang.

Ein vorsichtiger Beobachter mit einem Blick für Analogien wird dabei sehr bald erkennen, dass hier eine nicht übeschreitbare Genze vorliegt , die man nicht überwinden, sondern der man ausweichen muss, um auf ÜLG zu kommen.

Wer zügig und glatt aus einem Gefängnis herauskommen will, geht nicht durch die Wand, sondern nimmt die Tür, besonders dann, wenn die Wände undurchdringlich sind.
Um die physikalische Struktur der Lichtgeschwindigkeit zu entschlüsseln ,hat man den gesamten Apparat der Wellen-Erscheinungen, insbesondere der Schallwellen zur Verfügung.

Die Parallelen zwischen Schallausbreitung (in Gasen) und der Lichtausbreitug im Vakuum sind derart aufdringlich, dass man schon sehr verbohrt sein muss, diese nicht heranzuziehen, um sie für eine Beschreibung der ursächlichen Grenzwertbedingungen der LG zu verwenden.

Wer sich also von dem Zwang lösen kann, die Lichtgeschwindigkeit linear erreichen zu wollen, sollte sich schleunigst um diese Parallelen kümmern und eine Struktur-Transformation von den Schallwellen auf die Lichtwellen wagen.

Die Lösung ist dann nicht mehr weit und kann mit einer Grenzwertverschiebung des c-Wertes auf n mal c erreicht werden, das heisst alle Schwierigkeiten im Grenznahen Bereich der LG tauchen erst mit dem Wert nc wieder auf und das ist bereits vielfache ÜLG.

33wabiswalter@bislins.ch (Walter Bislin, Autor dieser Seite) 28.10.2017 | 23:34

Lichtgeschwindigkeit kann nicht mit Schallgeschwindigkeit verglichen werden

Werner: Die Parallelen zwischen Schallausbreitung (in Gasen) und der Lichtausbreitug im Vakuum sind derart aufdringlich, dass man schon sehr verbohrt sein muss, diese nicht heranzuziehen, um sie für eine Beschreibung der ursächlichen Grenzwertbedingungen der LG zu verwenden.

Lichtgeschwindigkeit kann nicht mit Schallgeschwindigkeit verglichen werden!

Wie hoch die gemessene Schallgeschwindigkeit ist, hängt vom entsprechenden Beobachter ab. Bewegt sich der Beobachter in der Luft, misst er eine andere Geschwindigkeit als wenn er sich gegenüber der Luft nicht bewegt. Bewegt er sich z.B. mit Schallgeschwindigkeit durch die Luft von der Schallquelle weg, bleibt ein Schallimpuls, der in seine Richtung ausgesandt wird, aus seiner Sicht stehen (Geschwindigkeit Null), da ihn der Impuls nie einholen wird. Bewegt er sich mit Überschall hat der Impuls sogar eine negative Geschwindigkeit, er bewegt sich quasi von ihm weg. Umgekehrt, wenn sich der Beobachter auf die Schallquelle zu bewegt, erreicht ihn der Impuls schneller, als wenn er still steht. Der Impuls hat von ihm aus gesehen dann eine höhere Geschwindigkeit als Schallgeschwindigkeit.

Aufgrund dieser Eigenschaft kann der Beobachter herausfinden, ob er sich gegenüber der Luft bewegt oder nicht, auch wenn er so von der Luft abgeschirmt wird, dass er seine Relativbewegung in der Luft nicht messen kann. Alleine durch die von ihm aus gemessene Geschwindigkeit eines Impulses in die eine und dann in die andere Richtung kann er die Schallgeschwindigkeit messen und seine Geschwindigkeit gegenüber der Luft bestimmen. Es gibt ein bevorzugtes Bezugssystem, in dem die Luft in Ruhe ist.

Lichtgeschwindigkeit hingegen ist für alle Beobachter immer gleich c, egal wie schnell sie sich gegenüber einer Lichtquelle bewegen. Das ist nicht intuitiv, aber Fakt. Licht braucht kein Medium zur Ausbreitung (kein sog. Äther). Daher gibt es auch kein absolutes Bezugssystem in ein Äther in Ruhe wäre. Alle unbeschleunigten Bezugssysteme sind gleichwertig. Man kann nur Relativbewegungen zwischen Bezugssystemen messen. In jedem Bezugssystem ist die gemessene Lichtgeschwindigkeit gleich c, egal wie schnell das Bezugssystem ist und egal wie schnell sicht die Lichtquelle bezüglich dieses Bezugssystems bewegt.

Lichtgeschwindigkeit ist keine Geschwindigkeit im Sinne einer Höchstgeschwindigkeit, die man irgendwie umgehen kann. Es ist die Geschwindigkeit, mit der sich kausale Zusammenhänge ausbreiten. Ich nenne sie ab jetzt Kausalgeschwindigkeit.

Kausalgeschwindigkeit ist ein Phänomen, das sich aus der Eigenschaft ergibt, wie Raum und Zeit zusammenhängen. Wir messen eigentlich die Kausalgeschwindigkeit (Lichtgeschwindigkeit) falsch. Sie erscheint uns nur als Limite, weil wir Raum und Zeit nicht als Einheit erfahren, sondern als getrennte Dinge. Alle physikalischen Messungen mit hohen Geschwindigkeiten zeigen jedoch eindeutig, dass Raum und Zeit nur verschiedene Projektionen der Raumzeit-Einheit sind.

In der Raumzeit wird Geschwindigkeit besser als Rapidität gemessen. In dieser Einheit ist die Kausalgeschwindigkeit unendlich und kann daher nicht überschritten werden. Nur für uns Wesen, die Raum und Zeit getrennt erfahren, erscheint diese Geschwindigkeit als eine Grenzgeschwindigkeit. Die Eigenschaft, dass Kausal-Rapidität nicht überschritten werden kann und dass sie für alle Beobachter immer gleich schnell ist, weil sie unendlich gross ist, muss auch in unserer Realität erhalten bleiben. Da wir Raum und Zeit als getrennt erleben, definieren wir Geschwindigkeit = Raum durch Zeit. Wenn wir Raum und Zeit als Raumzeit erleben könnten, wäre diese Definition von Geschwindigkeit unsinnig. Wir würden die unsere Kausalgeschwindigkeit als Rapidität definieren, sie keine Limite hat.

Siehe auch mein Beitrag weiter oben: Lichtgeschwindigkeit als Konsequenz der Lorentz-Transformation

Paradoxa bei Überlichtgeschwindigkeit

Wenn wir die Kausalgeschwindigkeit irgendwie umgehen könnten, würden Paradoxa entstehen. Zum Beispiel: Nehemen wir an zwei Raumschiffe A und B entfernen sich voneinander mit Unterlichtgeschwindigkeit. Wir betrachten die folgende Szene nur aus der Perspektive von Raumschiff B. Um zu sehen was da passiert, kann man sog. Minkowski-Diagramme verwenden.

Raumschiff B bekommt ein Problem und bittet Raumschiff A mit Überlichtkommunikation um Hilfe. Raumschiff A entsendet kurze Zeit daruf ein Rettungsschiff mit Überlichtgeschwindigkeit. Von Raumschiff B aus gesehen sieht das dann zeitlich wiefolgt aus (mit Überlichkommunikation betrachtet): Das Rettungsschiff taucht wie aus dem Nichts bei Raumschiff B auf und verdoppelt sich augenblicklich am selben Platz. Das eine Rettungsschiff bleibt bei B, während das andere sich nach A bewegt, wobei alle Abläufe in diesem Rettungsschiff von B aus gesehen zeitverkehrt verlaufen. Je nach Relativgeschwindigkeit zwischen den Raumschiffen A und B können sogar Situationen entstehen, wo das noch nicht angeforderte Rettungsschiff bereits auftaucht, aber nur, wenn es später auch wirklich angefordert wird.

Solche Dinge sind logisch unmöglich und können auch nicht passieren solange die Kausalgeschwindigkeit nicht überschritten werden kann.

34Werner Schneider 28.10.2017 | 14:12

Überlichtgschwindigkeit mit der Verschiebung eines qualsi- starren (steifen) Körpers

Es wird oft behauptet, die Schallgeschwindigkeit in Festkörpern sei deutlich kleiner als die Lichtgeschwindigkeit und es hätte dann keinen Sinn, z.B einen relativ starren Stab so zu verschieben, dass die Lageänderung an der Stirnseite (Stossaufnahme-Fläche des Stabes) in Vergleich mit der Lageänderung an der Rückseite (Stossabgabe-Seite) eine Überlichtschnelle Information längs des Stabes überträgt.

Unter einen Kraftstoss verstehen wir dabei eine kurzzeitige Krafteinwirkung der Form (p= Ft) auf einen Beweglichen aber trägen Massenkörper.

Dabei tritt nun folgende Besonderheit auf:

a) Ist der Probestab horizontal und reibungsfrei gelagert, so genügt eine Stosszeit, die unter der Reaktionszeit des Stabes liegt, um eine Schallwelle im Stab auszulösen - der Stab als ganzes reagiert auf grund seiner Trägen Masse nicht und eine Verschiebung seines Schwerpunktes geht gegen Null.

b) Ist der Stab horizontal gelagert und mit hilfe einer Aufhängung ein kleines Stück aus der
Ruhelage gezogen, dann kann die Längs-Bewegung des Stabes stossfrei eingeleitet werden, in dem man den Rückhalte-Faden trennt.(Informations-Eingang) Für diesen Fall wird der Stab inklusive jedes seiner restelastischen Moleküle von den Kraftlinien des Erdschwerefeldes erfasst und als ganzer Körper samt Schwerpunkt bis zum Anschlag gezogen. Eine nennenswerte Inhomogenität der Gravitativen Beschleunigungskräfte tritt nicht auf, eine Schallwelle kommt nicht zu stande.

Für diesen Fall (b) tritt dann eine Informationsübertragung Im Vergleich der
Stirnflächenverlagerung mit vielfacher Überlichtgeschwindigkeit ein und wiederlegt damit
die Theorie von der Konstanz der Lichtgeschwindigkeit in allen Bezugssystemen.

Wie ist das möglich?

Wir wissen ja bereits, dass die Träge Masse oder ihre Relativität die Grenzgeschwindigkeit (c) bestimmt. Schalten wir nun den Einfluss der Trägheit bei der Informationsübertragung aus, dann muss
sich das abgesendete Signal zwangsläufig mit unbegrenzter Geschwindigkeit (nur begrenzt
von den Messfehlern des Übertragers) ausbreiten.

Beim horizontal aufgehängten quasi-starren Metallstab begrenzter Länge wird die Träge Masse oder ihr Trägheitswiderstand dadurch gegen Null gesetzt, dass die Beschleunigungskraft nicht nur an der Frontseite des Stabes angreift, sondern an jedem einzelnen Molekül seines Kristallgittters. Dadurch reduziert sich die wirksame Trägheit des Ganzen Stabes auf die Trägheit eines Moleküls, wobei ein Druckaufbau zwischen den Stabenden vollkommen vermieden wird. Der Stab reagiert daher nicht mehr träge, sondern gravitativ und für die Gravitation gibt es keine nachweisbare Begrenzung der Ausbreitungsgeschwindigkeit.

Bei der Informationsübertragung der Stirnflächen-Positionen kommt es auf die Schwerpnktgeschwindigkeit
des Übertragers also Stabes nicht an, sondern nur auf die Verlagerungszeitpunkte seiner Endflächen.

35wabiswalter@bislins.ch (Walter Bislin, Autor dieser Seite) 28.10.2017 | 23:34

Schall- oder Lichtgeschwindigkeit in einem Material kann mit keinem Trick überwunden werden

Werner Schneider: Für diesen Fall (b) wird der Stab inklusive jedes seiner restelastischen Moleküle von den Kraftlinien des Erdschwerefeldes erfasst und als ganzer Körper samt Schwerpunkt bis zum Anschlag gezogen.

Es tritt dann eine Informationsübertragung Im Vergleich der Stirnflächenverlagerung mit vielfacher Überlichtgeschwindigkeit ein und wiederlegt damit die Theorie von der Konstanz der Lichtgeschwindigkeit in allen Bezugssystemen.

Falsch! Der Stab wird nicht als ganzes von der Schwerkraft in einem Moment verschoben. Wenn das Faden am einen Ende bei A durchtrennt wird, fehlt diese Haltekraft und die Moleküle dieses Endes beginnen sich zu beschleunigen. Da sich die Information, dass der Faden durchtrennt wurde, nur mit Schallgeschwindigkeit des Materials weiter pflanzen kann, weil jede Kraftänderung zwischen Molekülen nur mit Schallgeschwindigkeit sich von einer Gruppe zur nächsten fortpflanzen kann, wissen weiter entfernte Moleküle noch nichts davon. Sie werden von den näher bei A liegenden Molekülen aufgrund deren inertialen Masse immer noch auf ihrer Position gehalten.

Vielleicht mache ich gelegentlich eine Simulation um dies zu demonstrieren. Bis dahin verweise ich gerne auf einen Versuch mit einem Slinky, bei dem ein ähnlicher Fall vorliegt. Schau dir die folgenden Videos an:

Man sieht, dass das untere Ende der Feder (kann als Model für einen langen Stab herhalten) in der Luft an derselben Stelle verharrt, wenn das obere Ende losgelassen wird, bis die ganze Feder an diser Stelle angekommen ist. Die Erklärung ist, dass das untere Ende die Information, dass das obere Ende losgelassen wurde, erst erfährt, wenn die ganze Feder bereits dort angekommen ist, weil sich in der Feder ein Stoss langsamer ausbreitet als die Fallgeschwindigkeit des oberen Endes. Das untere Ende erfährt daher aufgrund der Massenträgheit benachbarter Windungen immer noch die Haltekraft gegen die Gravitation, auch wenn das obere Ende schon lange losgelassen wurde.

Seit mehr als 100 Jahren wird immer wieder versucht, die Theorie der Konstanz der Lichtgeschwindigkeit zu widerlegen. Bisher erfolglos. Es hat sich immer gezeigt, dass in den Versuchen Überlegungsfehler vorhanden waren oder dass mit physikalisch unmöglichen Gedankenexperimenten argumentiert wurde. Es macht aber keinen Sinn, physikalische Tatsachen mit physikalisch unmöglichen Gedankenexperimenten widerlegen zu wollen. Gedanklich ist alles möglich, in der Realität aber nicht.

36Werner Schneider 29.10.2017 | 13:25

Es kommt wie bei allen Theorien auf die konsequente Einhaltung der Modell-Prämissen an und da kann
es schon mal passieren, dass man Fünfe gerade lässt.

Auch wenn schon viele Versuche zur Überwindung des c-Postulats über die Bühne gegangen sind,
so ist das kein Beweis dagegen, dass es doch mal einer schafft...

Die Träge Masse der Stabmoleküle sitzt in elastischen Teilchen mit Oberflächenspannung, diese
Massenfreie feldbedingte Spannung leitet die Haltekraft am Anfang des Stabes von Punkt zu Punkt weiter, wobei zu überlegen ist, die Thermische Unruhe der Moleküle durch Tiefkühlung zu minimieren.
Wir haben also von einer massenfreien bzw Trägheitsfreien Feldkraftkette auszugehen, und dürfen nicht den Fehler machen, in dieser Reihenschaltung eine STATISCH wirksame oder verspätet Dynamisch wirkende Träge Masse dominieren zu lassen.
Jeder Träge Massenpunkt wird mit seiner Oberflächen-Spannung in Reihe überbrückt.

Der Unterschied:
Träge Masse wird induktiv durch Relativ-Beschleunigung frei - Feldspannung reagiert Trägheitslos!

Die Folge ist klar: Bei einem Wegfall der Vorspannung am Haltehaken des Stabes, gehen alle beteiligten
horizontalen Vorspannungen der Kette unverzögert auf Null und werden von den erdbedingten Gewichtskräften vertikal und.gleichzeitig nach unten gezogen und das mit einem minimalen Messfehler der Übertragung.

Das habe ich schon vor ca 50 Jahren in allen wesentlichen Teilen geprüft.
Niemand - ausser den Rettern der Einstein-Relativität hat das bisher zu kippen vermocht.
Es kam ihnen immer die gleiche lückenhafte Logik der Grosszügigkeit dazwischen.

Trotzdem warte ich gern ab, welche Lücken Ihre Simulation zu Tage fördert.

37Werner Schneider 29.10.2017 | 15:34

Nachtrag-

Unter Beachtung der Grundlagen darf man in der SRT keinen Starren Körper einführen, weil die Materie grundsätzlich
elastisch (Biegsam) aufgebaut ist. (MAX v. LAUE)
Nun das macht garnichts, wenn wir das Hooksche Gesetz beachten.

Vorspannugs-Axiom des Starren Körpers
Setzt man einen elastischen Körper in der gewünschten Bewegungsrichtung unter eine
Vorspannung, die geringer ist, als seine Steifigkeit - dann wird er wie ein ideal starrer Körper
beschleunigt, wenn man die Vorspannung aufhebt.
Das gilt natürlich auch für Schwere Körper, deren Schwerpunkt als Angriffspunkt aller
äusseren G-Krafte genommen werden kann. (I.NEWTON)
Ist zwar alles schon lange her, gilt aber heute noch.

38Werner Schneider 30.10.2017 | 11:59

Die Grenze der Relativitäts-Theorie

SRT und ART Einsteins gehen zunächst von einer Welt der elastischen Körper und Medien aus
und beziehen die Konstanz der Lichtgeschwndigkeit mit dieser Einschränkung auf alle Ereignisse
und Vorgänge in der Physik.

Das macht zunächst einen sehr allgemeingültigen Eindruck, verkennt aber die Tatsache, dass
wir in einer Dualen Welt leben, wo jeder Begriff der Dinge in zwei konträren Varianten von
Satz und Gegensatz, von Teil und Gegenteil gedacht werden kann und über seine Existenz
des Denkbaren auch seine Existenz in der Wirklichkeit nahe legt.

Einer Welt der elastischen Körper darf man folglich dual ergänzend eine Welt der starren
Körper zuordnen und erwarten das beide Formen sich kompatibel ergänzen.

von LAUE sagt zwar in seiner Fassung der Einstein-Relativität im ersten Band seines
Buches "Die Spezielle Relativitätstheorie" bei Vieweg, Seite 42 u.a.:

Zitat:
Die Annahme eines Starren Körpers ist in der Relativitätstheorie unzulässig

liefert damit aber auch sofort das Gegenmittel zur Überwindung der Konstante c
nach der Anleitung:
Willst Du die Lichtgeschwindikeit der elastischen Medien übertreffen, führe einen
Starren Körper ein und realisiere die damit gewonnene Überlichtgeschwindikeit,
in dem Du die praktischen Voraussetzungen für die Darstellung des Experimentes
aus den vorhandenen Mitteln konstruierst.

Oder schaue Dich in der Natur um, wo Du Verhältnisse vorfindest, die dem Starren
bzw verschiebestarren Körper entsprechen.

Mit dieser Vorgehensweise ist es dann relativ einfach einen Starren Stab oder ein
starr verschiebbares bereits voll ausgedehntes Kraftfeld einzuführen und auf seine Eignung zu prüfen.

39Werner Schneider 30.10.2017 | 17:04

Grundlagen-Versuch zum Starren Körper

Ein starrer Körper kommt in der Natur nicht vor - sondern existiert dort nur in angenäherter Form
den letzten Schritt zu seiner Verwirklichung erfolgt über bestimmte Randbedingungen oder technische
Manipulation.

Das ist auch der Grund dafür, warum er nicht in der Einstein-Relativität berücksichtigt wird.
Entscheidend ist dabei das mechanische Prinzip der elastischen Elemente-Kopplung.
Diese kann zwischen den Abstufungen weich und hart schwanken, wobei sich nicht ausschliessen
lässt, dass ein Grenzfall existiert, der die Elastizität gegen Null gehen lässt.

Um ein Beispiel für eine verschiebestarre Übertragung von Information zu geben machen wir einen Versuch.

Wir verschieben eine Kunstoff-Marke (in Form eines Aufklebers) mit hilfe eines Gummifadens.
Dazu schneiden wir einfach den Ring eines sog. Flitsch-Gümmis auf und erhalten damit einen sehr weichen
und flexiblen Gummifaden.
Dieser wird einfach der Länge nach in Position gebracht(in derTisch-Ebene) und das eine Ende als
Informations-Eingang benutzt und andere Ende mit der Marke beklebt, die den Informations-Ausgang
repräsentiert.

Übertragungs-Vorgang 1:
Wir fassen den Faden am Eingang und schieben in Richtung der Marke.
Ergebnis:.
Der Faden faltet und kräuselt sich, die Marke bleibt in Ruhe.

Übertragungsvorgang 2
Wir spannen den Faden in einer gegabelten Haltevorrichtung ein und spannen ihn mit einer
entsprechend kleinen Zugfeder vor, die wir direkt an der Marke anschliessen und auf der Gabelseite1
befestigen - den Anfang des Fadens verbinden wir mit einem Stift, der auf der gegenüberliegenden
Gabelseite 2 verschiebbar eingespannt ist.

Die Vorrichtung ist bereit, wenn der Stifft soweit zurückgezogen und fixiert ist, dass der Gummifaden
deutlich gestreckt im Raum schwebt.
.

Verschieben wir nun den Stift ein kleines Stück in Richtung der Marke, ohne ihn ganz frei zu geben,
dann verschieben wir auch den dortigen Anfang des Fadens und dieser verschiebt dannt die Marke am
anderen Ende unverzüglich die gleiche Meldestrecke mit und zwar ohne jede Verzögerung.
Das entspricht einer Informations-Übertragung mit vielfacher Überlichtgeschwindigkeit.

Erst wenn man den Stift am Informations-Eingang so schnell verschiebt, dass die Vorspannfeder nicht
mitkommt, hat man die synthetische Steifigkeit des Fadens aufgehoben und er kräuselt sich wieder -
die Übertragungsgeschwindigkeit liegt dann weit unter der Lichtgeschwindikeit - es gelten wieder die Gesetzte
des elastischen Körpers.

 

.

40Walter 30.10.2017 | 18:50

Das funktioniert nicht mit Überlichtgeschwindigkeit, sondern viel, viel langsamer. Bitte schau dir doch nochmals die oben verlinkten Videos mit dem Slinky an. Das ist auch eine Feder mit Vorspannung und sie ist gleichzeitig sogar noch unter dem Einfluss der Gravitation. Und trotzdem, wenn man das eine Ende loslässt, bewegt sich das andere Ende erst, wenn das eine Ende dort angekommen ist. Die Vorspannung spielt keine Rolle. Allein die Schallgeschwindigkeit eines Material bzw. die Fortpflanzungsgeschwindigkeit einer Störung ist massgebend. Und diese Geschwindigkeit ist allein von der Steifigkeit des Materials abhängig. Diese Schallgeschwindigkeit ist bei allen Materialien die wir kennen weit, weit unter der Lichtgeschwindigkeit. Die Bewegungsimpulse wandern langsam von Atom zu Atom, egal ob die Verbindungen zwischen den Atomen vorgespannt sind oder nicht. Selbst wenn du irgend ein masseloses Kraftfeld oder sowas darüber spannen könntest, würde sich die Änderung im Feld auch nur maximal. mit Lichtgeschwindigeit fortpflanzen. Siehe Gravitationswellen. Die Raumzeit ist extrem steif, daher sind Gravitationswellen so schwach, selbst wenn Schwarze Löcher kollidieren. Und trotzdem pflanzen sich Gravitationswellen "nur" mit Lichtgeschwindigkeit aus.

Es gibt keine Möglichkeit, die Lichtgeschwindigkeit zu überwinden, denn in der Raumzeit ist die Rapidität (Lichtgeschwindigkeit in Raumzeit-Einheiten) des Lichtes oder irgendwelchen masselosen Wellen unendlich. Und schneller als unendlich geht nun mal nicht. Das bedeutet für uns, die wir Geschwindigkeit als Weg durch Zeit definieren, dass schneller als c eben nicht möglich ist.

41Werner Schneider 30.10.2017 | 23:28

Sehr bedauerlich das ganze - und ich muss schon sagen, auch unverständlich.

Vor 50 Jahren wurde noch auf die Argumente eines Neuen Ansatzes eingegangen und im einzelnen gezeigt
wo die Einstein-Relativität zu berücksichtigen war, heute wird einfach die Schublade des Vorurteils aufgemacht.

Es hat also nur noch Sinn, ein vollständig Messtechnisch ausgerüstetes Experiment zu bringen, wo nur noch die
Messuhren zeigen, was tatsächlich zutrifft.

Die gebrachten Gegenargumente mit Schall -und Gravitatiionswellen sind durch die Bank nicht zutreffend und in ihrer Analogie gar nicht verstanden worden.
Auch der Begriff der Relativgeschwindigkeit wird nicht punktgenau auf die von mir gebrachte Anordnung übertragen, wo es gar nicht darum geht, wie schnell sich Schallwellen ausbreiten, sondern darum, ob sie überhaupt relevant entstehen bzw mitwirken oder nicht.
Die besagten Gravitationswellen, die mal bei Hannover gemessen wurden, können überhaupt nicht von der verwendeten Messeinrichtung instantane Gravitations-Schwingungen von trägen Gravitationswellen unterscheiden, denn Gravitationswellen müssten träge Masse transportieren, was sie nicht tun, mit Ausnahme der begleitenden elektromagnetischen Feldänderungen. Ein Gravitationsfeld enthält keine Trägen Feldelemente oder Trägen
Feldquanten.

Auch die Tatsache, das grundsätzlich ein möglichst starrer also steifer Körper gewählt werden muss, dessen
Restelastizität mit der gewählten Vorspannung zu beseitigen ist, wird ignoriert.
Und das, obwohl mein Beispiel nun tatsächlich eine geradezu krasse Umformung von weich auf hart also von weich elastisich auf Minimal-Elastisch in Bezug auf die VERSCHIEBUNG offenlegt.

Um es noch mal zu sagen, zwischen den vorgespannten Enden des Gummifadens findet keine Relativbewegung
mehr statt, der Faden ragiert STARR! während er vom Stift in der Gabel verschoben wird.
Thermisch bedingte Restschwankeungen verhindern lediglich das Ereichen einer Unendlich hohen instantanen
Übertragung - für vielfache Überlichtgeschwindigkeit reicht es allemal.
Vielleicht gibt es ja andere Leser oder Kommentatoren, die wirklich auf die Einzelheiten des zugegebener Massen
recht einfach gestalteten Versuchs eingehen können.

Lieber Walter - tut mir leid, Du bist nicht objektiv.
Es ist daher auch nicht möglich, auf die genaueren Einzelheiten des Versuchs einzugehen und
nocht elementarer zu zeigen, wo die Einstein-Relativität ihre Grenze hat..

Noch ein Hinweis zum horizontal aufgehängten Pendelstab
Dieser wird durch 2 drehare schräg stehende Gelenkstäbe gehalten, die Rückhaltekraft wird durch einen
Haken an den Gelenkstäben übertragen, die auf den Schwerpunkt des Stabes wirkende Vorspannung über das Erdschwerefeld führt nur zu vertikal gerichteten Spannungen im Atomgitter- beim Lösen des Halte-Hakens
sinkt der Schwerpunkt des Stabes auf einer Schiefen Ebene mit fast Fallgschwindigkeit ab, alle noch vorhandenen
Inneren Spannungen bauen sich in vertikaler Richtung ab, die Beschleunigungskräfte am Atomgitter des Stabes
sind individuell an jedes Atom gebunden, ein elastisch relvant mitwirkender Erffekt ist an der Horizontalbewegung
der Stab-Enden nicht beteiligt und kann im Grunde gar nicht angewendet werden.
Man befindet sich hier auch nicht mehr im Gültigkeitsbereich der Einstein-Relativität, sondern im Bereich der Instantan-Physik.
Ein Zwang, hiermit die Einsteinrelativität in Frage zu stellen, besteht daher gar nicht - mit Ausnahme des Universal-Anspruchs. Dieser wird ohnehin durch die Ausnahmebedingung des Starren Körpers schon in der Fachliteratur eingeschränkt bzw begrenzt.

Tut mir leid, wenn ich mich anfangs teilweise missverständlich ausgedrück habe - aber die Erinnerung hat so ihre Lücken, die sich nur verzögert schliessen.
.

42Werner Schneider 31.10.2017 | 16:58

Nachtrag bezogen auf den Titel :Überlichtgeschwindigkeit mit unelastischen Körpern

Warum sollte die Verwendung von unelastischen Körpern als Informations -und Energieübertrager nur
über die Anregung mit einer Stossfolge durchführbar sein?
Ein frontal einwirkender Stoss auf einen stabförmigen Übertrager führt doch zwangsläufig zur Ausbildung
einer Dichtestörung, die sich dann als wellenförmige Querschnittsänderung mit Schallgeschwindigkeit
fortpflanzt.
Steht dahinter eine pragmatische Überlegung, die einen beonders langen Übertrager ausnutzen will
oder ist die Idee, einfach den ganzen Übertrager (als Stab sehr begrenzter Länge) zu verschieben,
zu exotisch und wirklichkeitsfremd?

Walter sagt: Je steifer der Körper ist, um so höher die erreichbare Schallgeschwindigkeit, die aber letztlich
über die Einstein-Relativität begrenzt wird.
Ja, wer wird dann aber die Schallgeschwindigkeit benutzen oder riskieren wollen, wenn er eine überlichtschnelle
Übertragung anstrebt?

Es geht doch zunächst darum, zwischen 2 Punkten im Raum eine Brücke zu bilden, die unter Ausnutzung aller
physikalisch greifbaren Mittel und Gesetzmässigkeiten eine Verbindung herstellt, die der Einstein-Relativität
in entscheidendem Masse ausweichen kann, wenn die Übertragungs-Geschwindigkeit von Information und
Energie weit über der Lichtgeschwindigkeit liegen soll..
Dann gilt aber: Je steifer der Körper ist um so weniger verformt er sich, wenn man ihn als ganzen verschiebt.

Das ist doch das Motiv der praktikablen Überlichtgeschwindigkeit - aber doch nicht, auf eine fragwürdige Suche
zu gehen, um ein Teilchen aufzuspüren, dass sich nicht relativistisch beeindrucken lässt -
das wäre wohl zum grossen Teil nicht mehr von dieser Welt und tatsächlich wenig brauchbar
und damit nur ein vorläufiger Erfolg.

Mein Weg liegt nur darin, nicht alles daran zu setzen, die Gültigkeit des Relativitätsprinzips anzuzweifeln, sondern
es als eine Herausforderung zu betrachten, es kompatibel und nutzbar in den Apparat der Physik zu übernehmen
Und das bedeutet einfach nach Lösungen für die Vermeidung von Extremen zu suchen.

Am besten hat sich dabei die Methode bewährt, Grenzwerte, Barrieren und Schranken erst mal zu umgehen,
und das in Bezug auf die Lichtgeschwindigkeit als wirksamer Grenzwert, einfach mit der Verschiebung
des Grenzwertes zu versuchen.
Weil der wesentlich von der Trägen Masse oder besser dem gravitativ-induktiven Trägheitswiderstand
abhängt, dies mit einer Vermeidung oder einem Abbau dieses Trägheitswiderstands anzugehen.
Das ist immer einen Versuch wert.
Mit dem Pendelstab-Experiment habe ich mir diesen Versuch ausgedacht, vorher viel zum Thema Relativität
und Inertial-System, Gravitation und Träge Masse, Vektoren und Matrizen gelesen und vor allem nie die Verbindung
zur Elektrodynamik aus den Augen verloren.
Das hat mir geholfen, die Duale Gesamtheit der Physik zu erkennen.

43Bruno Amstad 15.11.2017 | 14:24

Hallo Walti, hier eine Frage die mich beschäftigt.

Da laut Relativitätstheorie (wenn ich das richtig verstanden habe), Beschleunigung in ihrer Wirkung nicht von Gravitation zu unterscheiden ist, würde das nicht bedeuten dass die Gravitation in einem schwarzen Loch, Überlichtgeschwindigkeit entsprechen würde? Oder ist dies die Singularität, da sich die beiden Kräfte die Waage halten?

Danke und lieber Gruss
Bruno

44wabiswalter@bislins.ch (Walter Bislin, Autor dieser Seite) 17.11.2017 | 22:26

Flugbahnen von Photonen in einem Schwarzen Loch

Bruno, Ich bin mir nicht sicher ob ich deine Frage verstanden habe. Willst du wissen, wie ein Photon, das innerhalb des Ereignis-Horizontes (EH) nach aussen geschickt wird und dabei ja Lichtgeschwindigkeit haben muss, trotzdem nach innen fliegen kann?

Nehmen wir an, ein Raumschiff durchfliegt den EH eines riesigen Schwarzen Lochs (SL). Vom Raumschiff aus gesehen ist der EH nichts besonderes. Wenn nun ein Photon nach aussen gesendet wird, muss es sich im Koordinatensystem des Raumschiffs mit Lichtgeschwindigkeit nach aussen bewegen. Bezüglich eines Koordinatensystems ausserhalb des SL kann es aber nur nach innen fliegen. Dann müsste doch das Raumschiff von aussen gesehen mit Überlichtgeschwindigkeit gegen das Zentrum des SL fliegen. Ist das nicht eine Verletzung der Spziellen Relativitäts-Theorie (SR)?

Nein, es widerspricht nicht der SR, die besagt, dass nichts schneller als mit Lichtgeschwindigkeit durch den Raum an der Stelle des Objektes fliegen kann. Der Raum selbst kann aber tun was er will. Im Falle eines SL bewegt sich der Raum innerhalb des EH von ausserhalb des SL gesehen mit Überlichtgeschwindigkeit auf das Zentrum des SL zu. Daher bewegt sich das Raumschiff, das relativ zum Raum an seiner Position sogar unbewegt sein kann, von aussen gesehen mit dem Raum mit Überlichtgeschwindigkeit ins Zentrum des SL. Das nach aussen gesendete Photon bewegt sich vom Raumschiff mit Lichtgeschwindigkeit in Richtung EH. Da sich aber der Raum schneller in Richtung Zentrum des SL bewegt, erreicht das Photon den EH nie. Von aussen gesehen bewegt es sich immer auf das Zentrum des SL zu - alles innerhalb des EH bewegt sich immer ins Zentrum des SL.

Das Ganze kann man sich vielleicht besser ausserhalb eines SL vorstellen. Wie du weisst, expandiert das Universum. Das bedeutet, dass je weiter entfernt ein Objekt von uns ist, umso schneller entfernt es sich von uns, obwohl es sich nicht durch den Raum an seiner Position bewegt, sondern der Raum selbst expandiert und entfernt sich von uns mit dem darin still stehenden Objekt. Je weiter weg ein Raumpunkt von uns entfernt ist, umso schneller bewegt sich also dieser Raum von uns weg mit allen darin still stehenden Objekten.

Nun gibt es eine Entfernung, ab der sich der Raum mit Überlichtgeschwindigkeit von uns entfernt. Das ist keine Verletzung der SR, denn nichts bewegt sich mit Überlichtgeschwindigkeit durch den Raum. Die Kugelschale in dieser Entfernung wird als kosmischer Ereignishorizont bezeichnet. Alles was innerhalb dieses EH liegt, wird als das sichtbares Universum bezeichnet. Alles was ausserhalb liegt, kann uns nie erreichen, selbst Licht nicht. Es ist also eine analoge Situation wie beim SL. Ein Raumschiff, das ausserhalb dieses EH fliegt oder still steht bezüglich seines lokalen Raums, der sich von uns aus gesehen mit Überlichtgeschwindigkeit entfernt, kann nun einen Lichtstrahl in unsere Richtung senden. Von ihm aus gesehen fliegt dieser Lichtstrahl mit Lichtgeschwindigkeit in unsere Richtung. Von uns aus gesehen bewegen sich aber Lichtstrahl und Raumschiff von uns weg, das Raumschiff sogar mit Überlichtgeschwindigkeit. Das Licht kann unseren EH und damit uns nie erreichen. Der Raum zwischen uns und dem Lichtstrahl wird schneller grösser, als die Strecke die das Licht in dieser Zeit zurück legt. Beachte, dass jeder Punkt im Universum seinen eigenen EH hat, denn alle Punkte sind gleichwertig, es gibt kein Zentrum.

Gravitation könnte man sich also als eine Art Raum-Fluss vorstellen, dessen Geschwindigkeit zunimmt, je näher man einer Masse kommt. Diese Geschwindigkeitszunahme ist nichts anderes als die Gravitations-Beschleunigung der Masse. Da der Raum-Fluss für alle Objekte derselbe ist, werden alle Objekte gleich stark im Raum-Fluss in Richtung Massenzentrum beschleunigt. Das gilt auch für einen Lichtstrahl, der nahe an einer Sonne vorbei fliegt. Da er sich sehr schnell, aber geradlinig im Raum-Fluss der Sonne bewegt, ist seine scheinbare Ablenkung nur gering, denn der Fluss bewegt sich in der Zeit des Vorbeifluges nur wenig. Die Flugbahn eines langsameren Körpers ist stärker gekrümmt, da er länger dem Raum-Fluss der Sonne ausgesetzt ist.

Dieses Modell des Raum-Flusses entspricht nicht der Realität. Der Raum-Fluss wäre etwa vergleichbar mit dem Licht-Äther. Es gibt nach der SR aber keinen solchen Äther und keine absolute Geschwindigkeit bezüglich eines lokalen Raum-Flusses. Es gibt nur Realtiv-Geschwindigkeiten bezüglich anderer Objekte und die Invarianz der Lichtgeschwindigkeit. Dennoch kann dieses Modell vielleicht ein paar Dinge klarer machen.

45Werner Schneider 22.11.2017 | 22:11

Der Raum an sich hat keine Substanz und kann eigentlich nur als ein geometrisches Gebilde oder Konstrukt
verstanden werden.
Darum sind Lichtwellen im Leeren oder freien Raum oder im Vakuum eigentlich nicht vorstellbar.
Führt man einen Träger oder ein Trägermedium für die Lichtwellen ein, bekommt man Widersprüche
zu den Gesetzen der Mechanik, die eine Masse des Trägers voraussetzen, die aber nicht nachweisbar ist.
(Lichtäther-Theorie)

Diese Schwierigkeiten lasssen sich umgehen, wenn man die Masse des Freien Raums oder Vakuums auf
die Masse der Lichtquellen reduziert und das Schwerefeld jeder Quelle als einen Dynamischen Transporter
auffasst, der selbst eine innere permanent expandierende Feld-Struktur aufweist, die einer isotropen Strömung
vom Quellenzentrum gleichkommt und zwar mit der Expansionsgeschwindigkeit v(exp)= c = const

Das vereinfacht das Problem des Leeren Raums auf ganz überraschende Weise:
Die Lichtquanten der Quelle brauchen dann nur bei entsprechender Anregungs-Energie aus dem Kristallgitter
gelöst und auf den Transporter des Schwerefeldkörpers gekoppelt werden und würden dann von dessen
Expansionsdynamik erfasst mit c(exp)=const davongetragen werden, bis sie entweder auf eine weitere Masse
mit m(o)>0 auftreffen und absorbiert oder reflektorisch wieder mit c=const von deren Oberfläche auf das zugehörige Schwerefeld umsteigen und weiter getragen werden.

Mit einem Schlag ist die Leere des Raums überwunden und dasLicht bewegt sich wieder "analog" zu den übrigen Wellenerscheinungen an Masse gebunden und mithilfe einesTrägers, der für jede Quelle aussserdem
ein eigenes Inertial-System festlegt.

Ich finde diese Lösung zumindest als Modellvorstellung so überzeugend, dass ich sie hier einmal
vorstellen möchte.

46AndiThoni 29.03.2018 | 23:41

Hallo @Wabis

Ein sehr guter Blog, wie ich finde.

Ich habe die verschiedenen Sichtweisen hier mit Interesse gelesen. Das Ganze ist einfach unfassbar faszinierend für mich.

Du schreibst zum Thema Schwarzes Loch folgenes:

"Alles was ausserhalb ... des kosmischen Ereignishorizontes ... liegt, kann uns nie erreichen, selbst Licht nicht."

Ist dies tatsächlich so der Fall?

Ich denke da folgendermaßen:

Stellen wir uns vor, zwischen dem sich mit Überlichtgeschwindigkeit von uns entfernenden kosmischen Ereignishorizont (Grenze des sichtbaren Universums) und der Erde wäre ein Gummiband gespannt, Welches natürlich in seiner Länge immer weiter gedehnt wird, seine Enden entfernen sich folglich auch mit Überlichtgeschwindigkeit von einander. Das Gummiband repräsentiert den sich überall gleichmäßig schnell, die Beschleunigung mal unberücksichtigt gelassen, ausdehnenden Raum (und das Gummiband hier ist als eindimensionale Analogie zum Luftballon, der aufgeblasen wird, zu sehen).

Jetzt stellen wir uns einen Lichtimpuls vor, der vom kosmischen Ereignishorizont aus startet und sich auf dem Gummiband "reitend" in Richtung Erde auf den Weg macht. In der Nähe des kosmischen Ereignishorizonts und auch sonst auf sämtlichen kleineren Intervallstrecken entfernen sich die Angangs- und Endpunkte der jeweiligen Intervallstrecken auf dem Gummiband mit einer Geschwindigkeit weit unterhalb der Lichtgeschwindigkeit voneinander. Der Lichtimpuls macht sich also gegenüber dem Startpunkt mit Lichtgeschwindigkeit auf dem Gummiband Richtung Erde auf den Weg. Zunächst entfernt sich der Lichtimpuls dennoch weiter von der Erde, da sich ja das Ende der Gummibandes am kosmischen Ereignishorizont mit Überlichtgeschwindigkeit von der Erde entfernt. Dennoch legt der Lichtimpuls unzweifelhaft pro Zeiteinheit t' eine definierte Strecke s' auf dem Gummiband zurück. Hatte der Lichtimpuls zu Beginn seiner Reise 100% der zunehmenden Länge des Gummibandes zur Überwindung vor sich, so sind es nach einer Zeit t' der Reise weniger als 100% der Länge des während der bis dahin absolvierten Reisezeit weiter an Länge zugenommenen Gummibandes als Wegstrecke, die dem Licht noch verbleiben, und dies unabhängig davon, wie schnell sich das Gummiband ausdehnt. Dem entsprechend verkürzt sich die noch zurückzulegende Strecke auf dem Gummiband prozentual immer weiter von unter 100% immer weiter abwärts.

Irgendwann, in Abhängigkeit der Ausdehnungsgeschwindigkeit des Gummibandes kommt der Lichtimpuls an einen Punkt, der sich genau mit Lichtgeschwindigkeit von der Erde entfernt, der Lichtimpuls steht für einen Moment scheinbar still, jedenfalls gegenüber der Erde. Ab diesem Punkt beginnt der Lichtimpuls sich der Erde zu näher, und zwar in Bezug auf die Erde erst ganz langsam und dann immer schneller. Bis er eines Tages mit gewohnter Lichtgeschwindigkeit auf der Erde ankommt.

Die prozentuale "Abarbeitung" der Länge des Gummibandes durch einen darauf Reisenden (hier das Licht) findet meiner Ansicht nach immer statt, unabhängig von der Ausdehnungsgeschwindigkeit des Gummibandes. Selbst eine Schnecke würde auf dem Gummiband rutschend irgendwann auf der Erde ankommen.

Wenn dies aber so wäre, würde uns auf der Erde dann nicht auch irgendwann jedes Lichtsignal des Universums erreichen, egal von wie weit entfernt es auch abgestrahlt würde?

Oder unterliege ich da einem Denkfehler?

47Rene Schleritzko 17.07.2018 | 19:16

Hallo an die Runde,

Bei dem Thema Lichtgeschwindigkeit muss ich immer anfangen zu Schmunzeln da es mich an die Zeit meiner Hauptschule erinnert.
Mathe und Physik Lehrer nahmen dieses Thema gemeinsam durch bis die Frage kam können sich Dinge schneller bewegen als Licht, stille, natürlich musste ich raus rufen (anders ging es bei mir nicht :-) "natürlich ist doch kein Ding".
Es brach eine fürchterliche Diskussion aus und ich fragte damals wenn Sie mir alles mir Ja beantworten habe ich Recht oder ? ich fing an das man im All ja sehr viel Platz hat also Raum kein Problem sein kann Antwort "Ja" gut sagte ich denn wir brauchen sehr viel Platz. meine zweite Frage war ob es Motoren gibt die sich mit 60U/min drehen, die Antwort war "Ja". meine dritte Frage war, können Sie sich einen Kreis mit 600.000 km Umfang vorstellen, Die Antwort, ein sehr großer Kreis aber "ja" können Sie mir den Durchmesser aus dem Kreisumfang berechnen, die Antwort war "JA" und ich darauf ganz Stolz, sehen Sie es können sich Dinge sogar mit zweifacher Lichtgeschwindigkeit bewegen wenn man nur genug Platz hat :-D

48wabiswalter@bislins.ch (Walter Bislin, Autor dieser Seite) 18.07.2018 | 23:02

@Rene: Die Umlaufgeschwindigkeit eines real konstruierbaren Rades kann niemals Lichtgeschwindigkeit erreichen. Atome sind durch elektrostatische Kräfte aneinander gebunden. Die Kraftwirkungen können sich also maximal mit Lichtgeschwindigkeit ausbreiten. Dies bedeutet, dass ein solches Rad auseinander fallen muss, bevor irgendwo auch nur annähernd Lichtgeschwindigkeit erreicht wird. Die Spezielle Relativitätstheorie begrenzt also gewisse physikalisch denkbaren mechanischen Konstruktionen schon prinzipiell. Siehe Ehrenfestsches Paradoxon.

49Werner Maurer 05.06.2019 | 06:29

Mit Kraft gleich Masse mal Beschleunigung anfangen und dann die relativistische Masse (Deus Ex Machina) ins Spiel bringen, heisst das Pferd vom Schwanz her aufzäumen. Die Frage ist doch, welche Gesetze in der Relativitätstheorie noch gelten. Das sind Impuls- und Energiebilanz, Geschwindigkeit ist Verhältnis von Impuls zu Masse, Leistung einer Kraft und die berühmte Einsteinsche Formel. Ich habe das in einem Video dargelegt
https://youtu.be/E7zp4mqI-cA

So eine Idee könnte man sogar an einer Berufsmaturitätsschule bringen.

50Peter Hager 25.09.2019 | 11:49

Fehlt hier nicht erst mal die Definition von "Geschwindigkeit"? Nach meinem Verständnis ist dies die Ortsänderung eines Gegenstandes relativ zu einem bestimmten Punkt. Lichtgeschwindigkeit wird aber in der Physik als absolute Größe bezeichnet. Wer dies akzeptiert, für den gibt es selbstverständlich auch einen Referenzpunkt, also jener imaginäre, absolut, unverrückbare Nullpunkt, zu dem sich das Licht und alles andere relativ bewegt. Wo der sich befindet, sagt aber keiner. Nicht mal dessen, zur Definition einer absoluten Geschwindigkeit unabdingbaren, Existenz wird erwähnt. Zumindest bislang habe ich noch nie von einem solchen fixen Zentrum des Raumes gehört oder gelesen. Damit wird in meinen Augen auch die Diskussion um jede "absolute" Geschwindigkeit gegenstandslos. - Nun behaupten Wissenschaftler, Lichtgeschwindigkeit würde, relativ zu uns, schon überschritten, aber dort würde sie eigentlich gar nicht überschritten, sondern nur der Raum würde "transportiert". So kann man es natürlich auch ausdrücken, wenn man einräumen muß, daß es keinen einzigen, absoluten Referenzpunkt gibt, sondern Geschwindigkeit immer nur relativ zu einem willkürlich ausgewählten Referenzpunkt ist. Der wird jetzt einfach als "transportierter Raum" bezeichnet. Ist doch gehuppt wie gesprungen! Und selbst dieser kann sich relativ zu uns mit vielfacher Lichtgeschwindigkeit bewegen. Erfahren werden wir das allerdings nie. und somit bleiben wir innerhalb jenes Ereignishorizontes stecken, den die Lichtgeschwindigkeit, relativ zu unserem eigenen Aufenthaltsort gemessen, uns setzt.

51wabiswalter@bislins.ch (Walter Bislin, Autor dieser Seite) 25.09.2019 | 16:17

Lichgeschwindigkeit ist Invariant

Peter, du hast die spezielle Relativitätstheorie im Allgemeinen und die Lichtgeschwindigkeit im speziellen nicht verstanden.

Lichtgeschwindigkeit ist nicht absolut bezüglich eines absoluten Referenzpunktes. Es gibt keinen absoluten Referenzpunkt. Das ist die erste beobachtbare Tatsache (das klassische Relativitätsprinzip), die zur speziellen Relativitätstheorie führte. Die zweite Tatsache ist, dass sich Licht in der Raumzeit immer gleich schnell ausbreitet, egal bezüglich was gemessen wird.

Der korrekte Ausdruck ist: Lichtgeschwindigkeit ist eine Invariante in der Lorentz-Transformation. Bei einem physikalischen System oder Vorgang beschreibt eine Lorentz-Invariante eine Eigenschaft, die von allen Inertialsystemen aus mit gleichem Wert beobachtet wird, wie z.B. die Lichtgeschwindigkeit c, die Masse m, die Teilchenzahl, die elektrische Ladung etc. Es gibt also noch weitere Eigenschaften neben der Lichtgeschwindigkeit, die für alle Beobachter gleich sind, und wir haben keine Mühe damit, dies zu akzeptieren.

Die Invarianz der Lichtgeschwindigkeit bedeutet somit, dass alle gleichförmig bewegten Beobachter immer dieselbe Lichtgeschwindigkeit in ihrem Bezugssystem messen, egal von wem das Licht ausgesandt wird. Wenn also beispielsweise Beobachter B sich mit v von Beobachter A entfernt und Beobachter A einen Lichtimpuls in die Richtung von B sendet, so bewegt sich dieser Lichtimpuls sowohl für Beobachter A als auch für Beobachter B mit Lichtgeschwindigkeit c bezüglich des jeweiligen und aller anderer Bezugssysteme! Du kannst also einen beliebigen Bezugspunkt wählen. Die gemessene Lichtgeschwindigkeit bezüglich jedes Bezugspunktes ist immer dieselbe. Das ist gemeint mit der Konstanz, der Absolutheit, besser Invarianz, der Lichtgeschwindigkeit.

Dies erscheint dem normalen Verstand völlig unmöglich, ist jedoch eine fundamentale Eigenschaft unseres Universums.

Beschrieben werden kann dies mit der Lorentz-Transformation und der Einführung der Raumzeit der speziellen Relativitätstheorie. Das bedeutet aber, dass wir den Begriff von absolutem Raum und absoluter Zeit aufgeben müssen.

Dies hat weitreichende Konsequenzen für alle physikalischen Gesetze sobald wir in Geschwindigkeitsregionen nahe der Lichtgeschwindigkeit kommen. Die von der klassischen Mechanik vertrauten einfachen physikalischen Gesetze, z.B. alle Newton'schen Gesetze, die von absolutem Raum und absoluter Zeit ausgehen, sind dann nicht mehr akkurat und müssen korrigiert werden. Die klassischen Gesetze können jedoch weiterhin als sehr gute Annäherungen angewandt werden, solange die involvierten Geschwindigkeiten weit unterhalb Lichtgeschwindigkeit liegen. Die klassischen Gleichungen erhält man aus den Gleichungen der Relativitätstheorie im Grenzfall von nicht-relativistischen Geschwindigkeiten.

Mit der speziellen Relativitätstheorie können diese Konsequenzen vorhergesagt und quantifiziert werden, z.B. Zeit-Dilation, Längen-Konraktion, E = m·c2 usw. Und diese Vorhersagen haben sich alle ausnahmslos bestätigt. Wir haben heute viele technische Anwenungen, die nur deshalb funktionieren, weil sie auf der Relativitätstheorie basieren. Um nur ein Beispiel zu erwähnen: GPS.

Die spezielle Relativitätstheorie und die konstanz der Lichtgeschwindigkeit lässt sich übrigens auch alleine aus beobachtbaren Symmetrien des Universums herleiten: Derivation of Special Relativity from basic observable Principles.

Nochmals: Es gibt keinen absoluten Bezugspunkt. Das wird schon mit dem klassischen Begriff des Relativitätsprinzip ausgesagt. Wenn es einen solchen Bezugspunkt gäbe, müsste dieser messtechnisch gefunden werden können. Dies aber würde voraussetzen, dass die physikalischen Gesetze von der Geschwindigkeit des Messlabors bezüglich des absoluten Bezugspunktes irgendwie abhängen würden. Anders wäre es nicht möglich den absoluten Punkt zu finden.

Wir stellen aber fest, dass in jedem gleichförmig bewegten Bezugssystem alle physikalischen Gesetzte absolut identisch sind, inklusive die des Elektromagnetismus. Fakt! Zudem sagen die Maxwell-Gleichungen des Elektromagnetismus eine nicht von der Geschwindigkeit des Bezugssystems abhängige (invariante) Lichtgeschwindigkeit c voraus: c = 1 / √ μ0 εo , wobei die Permittivität ε0 und die Permeabilität μ0 Naturkonstanten sind, die nicht von einer Geschwindigkeit, und somit von keinem Bezugssystem abhängig sind.

Überlichtgeschwindigkeit aufgrund der Expansion des Universums

Peter: Nun behaupten Wissenschaftler, Lichtgeschwindigkeit würde, relativ zu uns, schon überschritten

Lichtgeschwindigkeit ist eine lokale Geschwindigkeit. Das Licht bewegt sich in der lokalen Raumzeit invariant mit c. Dies gilt für jeden Ort im Universum. Die Allgemeine Relativitätstheorie besagt, dass Raumzeit sich biegen und dehnen kann (bestätigt durch viele Experimente). Wenn sich aber die Raumzeit selbst ausdehnt, kann sich ein weit entfernter Punkt im Universum mit beliebig hoher, auch mit Überlichtgeschwindigkeit von uns entfernen. Dies ist aber keine Verletzung der Konstanz oder der Grenze der Lichtgeschwindigkeit. Das Licht bewegt sich hier und dort immer noch mit c durch die lokale Raumzeit. Es ist also ein Unterschied, ob sich zwei nahe Beobachter relativ zu einander durch die selbe Raumzeit bewegen und die Lichtgeschwindigkeit messen, oder ob die Raumzeit zweier Beobachter weit ausanander liegt und sich dazwischen ausdehnt.

Wie kann man unterscheiden, ob sich etwas von uns entfernt, weil sich die Raumzeit ausdehnt oder weil sich etwas in der Raumzeit von uns entfernt?

Wir wissen dass sich nichts schneller als mit c durch den Raum bewegen kann, bestätigt durch viele Versuche. Wenn sich nicht die Raumzeit dehnt, sondern alles sich wie bei einer Explosion von einem Zentrum aus entfernt, könnte nichts schneller als c von diesem Zentrum aus weg fliegen. Das Universum würde in diesem Falle nicht homogen erscheinen. Alle Beobachtungen, insbesondere auch der kosmischen Hintergrundstrahlung, zeigen aber ein im grossen Bild extrem homogenes Universum ohne ein Zentrum. Diese Beobachtungen stimmen mit den Vorhersagen einer expandierenden Raumzeit überein.

Wir müssen uns mit der Tatsache abfinden, dass die Natur nicht so ist, wie wir sie uns gerne vorstellen. Zum Glück. Denn wenn die Natur so einfach wäre, wie wir uns dies gerne vorstellen (absoluter Raum und Zeit), dann würde Elektromagnetsimus nicht existieren, und somit würde es auch keine Atome geben, es würde uns nicht geben!

52badhofer 09.08.2022 | 10:14

Wenn sich Photonen oder Neutrinos parallel nebeneinander in die gleiche Richtung bewegen, sind sie dann relativ zueinander in Ruhe?

53wabiswalter@bislins.ch (Walter Bislin, Autor dieser Seite) 11.08.2022 | 22:30

Nein, Photonen sind masselos und bewegen sich mit Lichtgeschwindigkeit durch den Raum, wogegen Neutrinos eine sehr kleine Masse haben, und somit nie ganz Lichtgeschwindigkeit erreichen können.

Dein Kommentar zu diesem Artikel
Name
Email optional; wird nicht angezeigt
Kommentar
  • Name wird bei deinem Kommentar angezeigt.
  • Email ist nur für den Administrator, sie wird nicht angezeigt.
  • Du kannst deine Kommentare eine Zeit lang editieren oder löschen.
  • Du kannst Formatierungen im Kommentar verwenden, z.B: Code, Formeln, usw.
  • Externen Links und Bilder werden nicht angezeigt, bis sie der Admin freischaltet.

Auf dieser Seite

More Page Infos / Sitemap
Created Dienstag, 21. Februar 2012
Scroll to Top of Page
Changed Montag, 10. April 2017